Rocksolid Light

Welcome to novaBBS (click a section below)

mail  files  register  newsreader  groups  login

Message-ID:  

** MAXIMUM TERMINALS ACTIVE. TRY AGAIN LATER **


tech / sci.physics.relativity / Re: Special Relativity Fatal Error

SubjectAuthor
* Special Relativity Fatal ErrorTom Capizzi
+* Re: Special Relativity Fatal Errorrotchm
|+- Re: Special Relativity Fatal Errorwhodat
|`* Re: Special Relativity Fatal ErrorTom Capizzi
| +- Re: Special Relativity Fatal ErrorJanPB
| `* Re: Special Relativity Fatal Errorrotchm
|  `- Re: Special Relativity Fatal ErrorTom Capizzi
+* Re: Special Relativity Fatal ErrorJanPB
|+- Re: Special Relativity Fatal ErrorJ. J. Lodder
|`* Re: Special Relativity Fatal ErrorTom Capizzi
| `* Re: Special Relativity Fatal ErrorTom Roberts
|  +- Re: Special Relativity Fatal ErrorMaciej Wozniak
|  +* Re: Special Relativity Fatal ErrorTom Capizzi
|  |+* Re: Special Relativity Fatal ErrorRicardo Jimenez
|  ||`- Re: Special Relativity Fatal ErrorTom Capizzi
|  |+* Re: Special Relativity Fatal ErrorStan Fultoni
|  ||`* Re: Special Relativity Fatal ErrorTom Capizzi
|  || `* Re: Special Relativity Fatal ErrorStan Fultoni
|  ||  `* Re: Special Relativity Fatal ErrorTom Capizzi
|  ||   `- Re: Special Relativity Fatal ErrorStan Fultoni
|  |`* Re: Special Relativity Fatal ErrorTom Roberts
|  | +* Re: Special Relativity Fatal ErrorStan Fultoni
|  | |`* Re: Special Relativity Fatal ErrorTom Capizzi
|  | | +* Re: Special Relativity Fatal ErrorStan Fultoni
|  | | |`- Re: Special Relativity Fatal ErrorTom Capizzi
|  | | `* Re: Special Relativity Fatal ErrorStan Fultoni
|  | |  +* Re: Special Relativity Fatal ErrorTom Capizzi
|  | |  |`* Re: Special Relativity Fatal ErrorStan Fultoni
|  | |  | +* Re: Special Relativity Fatal ErrorTom Capizzi
|  | |  | |`* Re: Special Relativity Fatal ErrorStan Fultoni
|  | |  | | `* Re: Special Relativity Fatal ErrorTom Capizzi
|  | |  | |  `* Re: Special Relativity Fatal ErrorStan Fultoni
|  | |  | |   +* Re: Special Relativity Fatal ErrorTom Capizzi
|  | |  | |   |`- New crank? Old crank?Dono.
|  | |  | |   `* Re: Special Relativity Fatal ErrorTom Capizzi
|  | |  | |    `* Re: Special Relativity Fatal ErrorStan Fultoni
|  | |  | |     `* Re: Special Relativity Fatal ErrorTom Capizzi
|  | |  | |      +- Re: Special Relativity Fatal ErrorStan Fultoni
|  | |  | |      +- Re: Special Relativity Fatal ErrorMaciej Wozniak
|  | |  | |      +- Re: Special Relativity Fatal ErrorStan Fultoni
|  | |  | |      +- Re: Special Relativity Fatal ErrorMaciej Wozniak
|  | |  | |      +- Re: Special Relativity Fatal ErrorTom Capizzi
|  | |  | |      +- Re: Special Relativity Fatal ErrorTom Capizzi
|  | |  | |      +- Re: Special Relativity Fatal ErrorTom Capizzi
|  | |  | |      +- Re: Special Relativity Fatal ErrorTom Capizzi
|  | |  | |      +- Re: Special Relativity Fatal ErrorStan Fultoni
|  | |  | |      +* Re: Special Relativity Fatal ErrorMaciej Wozniak
|  | |  | |      |`* Re: Special Relativity Fatal ErrorJ. J. Lodder
|  | |  | |      | `* Re: Special Relativity Fatal ErrorMaciej Wozniak
|  | |  | |      |  `* Re: Special Relativity Fatal ErrorJ. J. Lodder
|  | |  | |      |   `* Re: Special Relativity Fatal ErrorMaciej Wozniak
|  | |  | |      |    `* Re: Special Relativity Fatal ErrorJ. J. Lodder
|  | |  | |      |     `* Re: Special Relativity Fatal ErrorMaciej Wozniak
|  | |  | |      |      `* Re: Special Relativity Fatal ErrorJ. J. Lodder
|  | |  | |      |       `* Re: Special Relativity Fatal ErrorMaciej Wozniak
|  | |  | |      |        `* Re: Special Relativity Fatal ErrorJ. J. Lodder
|  | |  | |      |         `- Re: Special Relativity Fatal ErrorMaciej Wozniak
|  | |  | |      +- Re: Special Relativity Fatal ErrorTom Capizzi
|  | |  | |      +- Re: Special Relativity Fatal ErrorStan Fultoni
|  | |  | |      +- Re: Special Relativity Fatal ErrorStan Fultoni
|  | |  | |      +- Re: Special Relativity Fatal ErrorMaciej Wozniak
|  | |  | |      +- Re: Special Relativity Fatal ErrorTom Capizzi
|  | |  | |      +- Re: Special Relativity Fatal ErrorStan Fultoni
|  | |  | |      +- Re: Special Relativity Fatal ErrorTom Capizzi
|  | |  | |      +- Re: Special Relativity Fatal ErrorStan Fultoni
|  | |  | |      +- Re: Special Relativity Fatal ErrorMaciej Wozniak
|  | |  | |      +- Re: Special Relativity Fatal ErrorTom Capizzi
|  | |  | |      +- Re: Special Relativity Fatal ErrorStan Fultoni
|  | |  | |      +- Re: Special Relativity Fatal ErrorTom Capizzi
|  | |  | |      +- Re: Special Relativity Fatal ErrorTom Capizzi
|  | |  | |      +- Re: Special Relativity Fatal ErrorStan Fultoni
|  | |  | |      +- Re: Special Relativity Fatal ErrorMaciej Wozniak
|  | |  | |      +- Re: Special Relativity Fatal ErrorTom Capizzi
|  | |  | |      +- Re: Special Relativity Fatal ErrorStan Fultoni
|  | |  | |      +- Re: Special Relativity Fatal ErrorTom Capizzi
|  | |  | |      +- Re: Special Relativity Fatal ErrorStan Fultoni
|  | |  | |      +- Re: Special Relativity Fatal ErrorMaciej Wozniak
|  | |  | |      +- Re: Special Relativity Fatal ErrorJanPB
|  | |  | |      +- Re: Special Relativity Fatal ErrorMaciej Wozniak
|  | |  | |      +* Re: Special Relativity Fatal ErrorTom Capizzi
|  | |  | |      |`* Re: Special Relativity Fatal ErrorRicardo Jimenez
|  | |  | |      | +- Re: Special Relativity Fatal ErrorRichard Hachel
|  | |  | |      | `- Re: Special Relativity Fatal ErrorTom Capizzi
|  | |  | |      +- Crank Tom Capizzi perseveresDono.
|  | |  | |      +- Re: Special Relativity Fatal ErrorStan Fultoni
|  | |  | |      +- Re: Crank Tom Capizzi perseveresTom Capizzi
|  | |  | |      +- Re: Special Relativity Fatal ErrorTom Capizzi
|  | |  | |      +- Re: Special Relativity Fatal ErrorStan Fultoni
|  | |  | |      +- Re: Special Relativity Fatal ErrorTom Capizzi
|  | |  | |      +- Re: Special Relativity Fatal ErrorStan Fultoni
|  | |  | |      +- Re: Crank Tom Capizzi perseveresDono.
|  | |  | |      +- Re: Crank Tom Capizzi perseveresTom Capizzi
|  | |  | |      +- Re: Special Relativity Fatal ErrorTom Capizzi
|  | |  | |      +- Re: Special Relativity Fatal ErrorJanPB
|  | |  | |      `- Re: Special Relativity Fatal ErrorMaciej Wozniak
|  | |  | `- Re: Special Relativity Fatal ErrorMaciej Wozniak
|  | |  `- Re: Special Relativity Fatal ErrorTom Capizzi
|  | `- Re: Special Relativity Fatal ErrorTom Capizzi
|  `* Re: Special Relativity Fatal ErrorThomas Heger
|   `* Re: Special Relativity Fatal ErrorMaciej Wozniak
|    `* Re: Special Relativity Fatal ErrorThomas Heger
+* Re: Special Relativity Fatal ErrorAldo
+* Re: Special Relativity Fatal ErrorSylvia Else
+* Re: Special Relativity Fatal ErrorThomas Heger
`- Re: Special Relativity Fatal ErrorJanPB

Pages:12345
Re: Special Relativity Fatal Error

<jgdlaaFfvmvU1@mid.individual.net>

  copy mid

https://www.novabbs.com/tech/article-flat.php?id=91554&group=sci.physics.relativity#91554

  copy link   Newsgroups: sci.physics.relativity
Path: i2pn2.org!i2pn.org!weretis.net!feeder8.news.weretis.net!3.eu.feeder.erje.net!feeder.erje.net!fu-berlin.de!uni-berlin.de!individual.net!not-for-mail
From: ttt_...@web.de (Thomas Heger)
Newsgroups: sci.physics.relativity
Subject: Re: Special Relativity Fatal Error
Date: Thu, 09 Jun 2022 09:20:45 +0200
Lines: 66
Message-ID: <jgdlaaFfvmvU1@mid.individual.net>
References: <b067e4e7-bb62-4130-950c-5663f4983806n@googlegroups.com> <jg38baFnm8cU1@mid.individual.net> <174bc459-0fc9-4de5-9bd2-de7f528dae07n@googlegroups.com>
Mime-Version: 1.0
Content-Type: text/plain; charset=utf-8; format=flowed
Content-Transfer-Encoding: 7bit
X-Trace: individual.net JG0bYAhnRxj066bu9LfXaAzmD7bc1P12kM+OMFgcSebCjP4xq6
Cancel-Lock: sha1:0CqqecnDVSyjChNH3efD6YM9kQE=
User-Agent: Mozilla/5.0 (Windows NT 6.0; WOW64; rv:31.0) Gecko/20100101 Thunderbird/31.4.0
In-Reply-To: <174bc459-0fc9-4de5-9bd2-de7f528dae07n@googlegroups.com>
 by: Thomas Heger - Thu, 9 Jun 2022 07:20 UTC

Am 08.06.2022 um 10:53 schrieb Tom Capizzi:
> On Sunday, June 5, 2022 at 4:38:06 AM UTC-4, Thomas Heger wrote:
>> Am 16.05.2022 um 03:48 schrieb Tom Capizzi:
>>> After years of rereading Einstein and arguing with his legions of cultists, I finally caught his unfixable mistake.
>> I had to cut off most of your message, because it is impossible to read
>> long texts without any formatting or white space.
>>
>> Ok, it is in fact possible to read a text, despite the author did
>> everything possible to make it hard.
>>
>> But you shouldn't count on the willingness to do so.
>>
>>
>> But let me try to answer your question, anyhow..
>>
>>
>> Einstein's worst error in 'On the electrodynamics of moving bodies' was,
>> that he attempted to create a function 'tau', which meant a coordinate
>> transformation between two coordinate systems in relative motion.
>>
>> Such coordinate transformations should leave the phenomenon described in
>> system A unchanged, if the desription is converted from A to coordinate
>> system B.
>>
>> But Einstein ascribed the function tau to the phenomenon itself, which
>> was interpreted as deformation of the form of that something (aka
>> 'length contraction').
>>
>> But coordinate transformations should not do that and change only the
>> description (iow: change the vectors, not the things, the vectors were
>> used to describe).
>>
>>
>> TH
>
> There is nothing intrinsically wrong with the idea of coordinate transformation.

Sure, coordinate transformations should be possible, because a
coordinate system is not a real thing, but a system which we use to
measure something.

Now a coordinate transformation should leave the thing we measure
unchanged, while alter measurement values.

> What Einstein got totally wrong was the definition of a valid measurement. He applied a standard that had been in use for centuries. Unfortunately for him, he was trying to describe the new physics of relativistic velocity. He took the valid mathematical procedure for determining parallelness and incorrectly assigned the low speed approximation of this mathematical standard to be his measurement protocol. A number of contradictions result, which can only be corrected by the addition of more ad hoc corrections. Reminds me of the failed attempts to correct the geocentric model of the Solar System by the addition of ever more epicycles. The geocentric model was ultimately abandoned because it was wrong. Special relativity deserves the same fate.
>

Einstein ascribed the measurement values to the thing we measure itself,
instead of to the measurement.

That is a terrible flaw.

E.g. the frequency change caused by the Doppler effect is 'unreal'
because it is only visible at the side of the observer.

Now Einstein's 'relativitic effects' (like length contraction and so
forth) were meant as real physical effect, which occur at the side of
the observed object, but were only justified by a coordinate transformation.

Such effects must not be ascribed to the attributes of the observed
thing, but to the observations.

TH

Re: Special Relativity Fatal Error

<1ccdd138-0756-49d9-b980-039996d3392fn@googlegroups.com>

  copy mid

https://www.novabbs.com/tech/article-flat.php?id=91585&group=sci.physics.relativity#91585

  copy link   Newsgroups: sci.physics.relativity
X-Received: by 2002:a05:620a:2903:b0:6a0:4d8f:8b88 with SMTP id m3-20020a05620a290300b006a04d8f8b88mr28231173qkp.328.1654805368835;
Thu, 09 Jun 2022 13:09:28 -0700 (PDT)
X-Received: by 2002:ad4:5bc8:0:b0:464:6079:d9d1 with SMTP id
t8-20020ad45bc8000000b004646079d9d1mr39256744qvt.24.1654805368574; Thu, 09
Jun 2022 13:09:28 -0700 (PDT)
Path: i2pn2.org!i2pn.org!usenet.blueworldhosting.com!feed1.usenet.blueworldhosting.com!peer01.iad!feed-me.highwinds-media.com!news.highwinds-media.com!news-out.google.com!nntp.google.com!postnews.google.com!google-groups.googlegroups.com!not-for-mail
Newsgroups: sci.physics.relativity
Date: Thu, 9 Jun 2022 13:09:28 -0700 (PDT)
In-Reply-To: <_bmdnXI-c7KTIT3_nZ2dnUU7_8zNnZ2d@giganews.com>
Injection-Info: google-groups.googlegroups.com; posting-host=209.6.152.121; posting-account=anpm0goAAAD7eq4-R7Tlsnov4nyr6Xqb
NNTP-Posting-Host: 209.6.152.121
References: <b067e4e7-bb62-4130-950c-5663f4983806n@googlegroups.com>
<5086313b-2d82-4e27-9aae-e1dd2cf7a954n@googlegroups.com> <60df787e-d27b-4f10-b7b2-03b3692ee97cn@googlegroups.com>
<_bmdnXI-c7KTIT3_nZ2dnUU7_8zNnZ2d@giganews.com>
User-Agent: G2/1.0
MIME-Version: 1.0
Message-ID: <1ccdd138-0756-49d9-b980-039996d3392fn@googlegroups.com>
Subject: Re: Special Relativity Fatal Error
From: tgcapi...@gmail.com (Tom Capizzi)
Injection-Date: Thu, 09 Jun 2022 20:09:28 +0000
Content-Type: text/plain; charset="UTF-8"
Content-Transfer-Encoding: quoted-printable
X-Received-Bytes: 9884
 by: Tom Capizzi - Thu, 9 Jun 2022 20:09 UTC

On Wednesday, June 8, 2022 at 11:11:18 AM UTC-4, tjrob137 wrote:
> On 6/8/22 3:13 AM, Tom Capizzi wrote:
> > On Monday, May 16, 2022 at 4:47:04 AM UTC-4, JanPB wrote:
> >> On Sunday, May 15, 2022 at 6:48:24 PM UTC-7, tgca...@gmail.com
> >> wrote:
> >>> After years of rereading Einstein and arguing with his legions of
> >>> cultists,
> >> Sigh. Yet another crackpot.
> >>
> >> -- Jan
> >
> > Yes. Don't you hate it when crackpot skeptics blather about how it is
> > impossible for there to be anything wrong with special relativity?
> Don't you hate it when people don't realize that the mathematics
> underlying Special Relativity has been proven to be as self-consistent
> as is Euclidean geometry, and as is real analysis?
>
> This essentially means that cranks and crackpots will not find
> inconsistencies in SR -- what it would take to do that is one of:
> A) finding an error in those proofs
> or
> B) finding internal inconsistencies in both Euclidean geometry
> and real analysis
>
> Nobody around here has any hope of doing any of that. Including you.
>
> > Einstein defined measurement [... claims of inconsistencies in SR]
>
> Such empty claims are useless. Show us what you think is a contradiction
> or internal inconsistency.
>
> > [...] A number of contradictions result, which can only be corrected
> > by the addition of more ad hoc corrections.
> There are no known inconsistencies in SR. There are no "ad hoc
> corrections" in SR. Your empty claims are useless -- SHOW US.
>
> Tom Roberts

Another poser pretending to be an authority.
Why don't you start by reading the whole post, and then responding to my argument instead of one of your own strawmen? Albert Einstein defined a legitimate indirect measurement as one that produces the same value as a direct measurement would. His own gedanken experiments, and every real experiment show that there is no protocol for indirect measurement above some threshold velocity that satisfies this requirement of validity. Yet he stumbles on with special relativity, using a protocol which does not meet his own standard of legitimacy. Then, he wills spacetime to behave like silly putty to make it agree with his version of mathematics. And the reason this works is because it restores the mathematics that belongs in the measurement protocol that his false assumption of what constitutes a valid measurement actually excludes. Because he put back exactly what he took out, the net result is the appearance of correct measurement prediction. Unfortunately, there is a price. Some would argue that there is no rule that the price must be paid, so they can accept either way. The price itself is the rejection of objective reality, a fundamental basis of science. More importantly, it is not that the existence of objective reality that matters here. It is the simple fact that a measurement of dilated time or contracted length does not agree with a direct measurement of the same interval. The fact that you can patch up this contradiction by corrupting the rules of logic does not alter the existence of the contradiction. But these are all negative comments. Hard to prove a negative.
So, I submit the correction. Now we can argue, and convince. The correct protocol for measurement is the Euclidean operation to measure parallelness, the dot product. In its algebraic form, it is routinely used to project components that are parallel and perpendicular to a velocity vector, since relativistic effects only occur in the direction parallel to motion. Normal components are invariant. In its geometric form, the internal composition is unimportant, because the dot product is the product of the magnitudes of the two arbitrary vectors with the cosine of the included angle between them. It is independent of the orientation of the coordinate system. The reference unit is the vector in the static frame, while the moving interval is the other vector being measured. In the usual relativity scenario, the two inertial frames have all their axes parallel. The angle is between a phase vector not included in the Minkowski 4. Its magnitude is defined by relative velocity, v = c sin(tilt). So two coordinate systems that are translating in parallel to each other are equivalent two spacetimes that are everywhere tilted in phase. Since translation is replaced by rotation, the two origins remain coincident, and the included angle is defined by the Law of Cosines, or directly by the relative velocity.
Now, construct two pairs of identical references for time and space intervals, verified by direct measurement. Then we place one pair in the moving frame and ask what they would measure from the static frame. From elementary geometry, ct' = ct cos(tilt) and r' = r cos(tilt). As long as cos(tilt) is non-zero, these equations can be rewritten as ct = ct' sec(tilt) and r = r' sec(tilt). But, using the definition of the Lorentz factor as a function of measured velocity, when v = c sin(tilt), γ = sec(tilt). So these equations reduce to ct = γct' and r = γr'. These are the Einstein equations of time dilation and length contraction, but NOTHING SHRANK.
These are geometric illusions that result from rotation of rigid objects into Elsewhere (which is apparently not confined to the volume of spacetime outside the past and future lightcones), where that portion can neither be seen nor measured. Once we incorporate all the intrinsic properties of hyperbolic trigonometry, special relativity becomes superfluous. After all, the dot product paradigm tells us never to expect to be able to measure more than a cosine projection. When we do the experiment, that is exactly what we measure, so the relativistic correction factor becomes unity. Special relativity evaporates.
So, the error in the proof that you demanded is that Einstein used a measurement protocol that was only valid at low speeds to analyze events that take place at relativistic velocities, contradicting his own assertion that valid indirect measurements are equal in magnitude to direct measurements. Rather than fix his mathematics, he willed spacetime to become plastic, and conned everybody, and I do mean EVERYBODY, by saying you have to buy the whole package, then it makes sense. Ha! Relativity only makes sense as a package. In fact, part of that package is the assertion that, as long as the numbers work, it doesn't matter if it doesn't make sense. They go further, and discourage anyone from looking for relativity to make sense. Geometric relativity does make sense and does not require corrupting any rules of logic to do so. And it isn't that this approach is elegant, which it is. It is supported by rigorous logic, whereas the Einstein Interpretation is not. Not only that, but the measurements defined by the Einstein Interpretation are not valid, because they do not agree with direct measurements of the same intervals. The geometric approach is valid at both extremes. The low-speed approximation is Newtonian physics, while the high-speed interpretation agrees completely with the experimental evidence.

Now, I am not interested in opinions about my use of terminology, or grammar, nor am I interested in opinions about the argument. This is not a peer-reviewed publication, and sometimes I get careless when composing these responses. But since my argument is based on the fact that special relativity is the result of a false premise, nothing derived from special relativity can be used as a counterargument, because that would be circular logic. Regurgitated dogma is therefore excluded. The only valid argument would be proving that the assumption leads to some kind of contradiction. To that extent, my argument is irrelevant. Unless you can prove that the Euclidean dot product protocol leads to a contradiction, my argument stands. And since it actually leads to the same predictions of measurement as the Einstein Interpretation, it seems unlikely that any new contradictions will pop out of the logic. It would require reality to contradict itself, which is basically impossible. After all, all those experiments already confirm all the numbers.

Re: Special Relativity Fatal Error

<n2t4ah5pr2ri9g1jj11gq4fnehbkcac55a@4ax.com>

  copy mid

https://www.novabbs.com/tech/article-flat.php?id=91590&group=sci.physics.relativity#91590

  copy link   Newsgroups: sci.physics.relativity
Path: i2pn2.org!i2pn.org!weretis.net!feeder6.news.weretis.net!news.misty.com!border2.nntp.dca1.giganews.com!nntp.giganews.com!buffer2.nntp.dca1.giganews.com!buffer1.nntp.dca1.giganews.com!nntp.earthlink.com!news.earthlink.com.POSTED!not-for-mail
NNTP-Posting-Date: Thu, 09 Jun 2022 17:35:01 -0500
From: ricky...@earthlink.net (Ricardo Jimenez)
Newsgroups: sci.physics.relativity
Subject: Re: Special Relativity Fatal Error
Date: Thu, 09 Jun 2022 18:35:04 -0400
Message-ID: <n2t4ah5pr2ri9g1jj11gq4fnehbkcac55a@4ax.com>
References: <b067e4e7-bb62-4130-950c-5663f4983806n@googlegroups.com> <5086313b-2d82-4e27-9aae-e1dd2cf7a954n@googlegroups.com> <60df787e-d27b-4f10-b7b2-03b3692ee97cn@googlegroups.com> <_bmdnXI-c7KTIT3_nZ2dnUU7_8zNnZ2d@giganews.com> <1ccdd138-0756-49d9-b980-039996d3392fn@googlegroups.com>
User-Agent: ForteAgent/8.00.32.1272
MIME-Version: 1.0
Content-Type: text/plain; charset=us-ascii
Content-Transfer-Encoding: 7bit
Lines: 7
X-Usenet-Provider: http://www.giganews.com
NNTP-Posting-Host: 71.190.237.81
X-Trace: sv3-vdIKFtRXmj3zs1Jqx4rCJfjBJCZDTqjOwZTBU+4PBZyZnv44MJpx47jOUpChS0Fi23KLfoTXKfjpoul!3IJuSAKP/zLj65qc0E4Lt/bkzmWB4cDbKyutmxOie9xIkksKw99EW7WUI7DO/jQsOREwjVCwbdeB!W3QSmQqJujel/1OaV1hEUbYVqRfE
X-Abuse-and-DMCA-Info: Please be sure to forward a copy of ALL headers
X-Abuse-and-DMCA-Info: Otherwise we will be unable to process your complaint properly
X-Postfilter: 1.3.40
X-Original-Bytes: 1871
 by: Ricardo Jimenez - Thu, 9 Jun 2022 22:35 UTC

On Thu, 9 Jun 2022 13:09:28 -0700 (PDT), Tom Capizzi
<tgcapizzi@gmail.com> wrote:

>So, I submit the correction. Now we can argue, and convince. The correct protocol for measurement is the Euclidean operation to measure parallelness, the dot product. In its algebraic form, it is routinely used to project components that are parallel and perpendicular to a velocity vector, since relativistic effects only occur in the direction parallel to motion. Normal components are invariant.

Are you assuming that the usual Pythagorean theorem is valid in the
(x,t) plane rather than the hyperbolic one?

Re: Special Relativity Fatal Error

<jgfe9tFpcloU1@mid.individual.net>

  copy mid

https://www.novabbs.com/tech/article-flat.php?id=91591&group=sci.physics.relativity#91591

  copy link   Newsgroups: sci.physics.relativity
Path: i2pn2.org!i2pn.org!weretis.net!feeder8.news.weretis.net!3.eu.feeder.erje.net!feeder.erje.net!fu-berlin.de!uni-berlin.de!individual.net!not-for-mail
From: syl...@email.invalid (Sylvia Else)
Newsgroups: sci.physics.relativity
Subject: Re: Special Relativity Fatal Error
Date: Fri, 10 Jun 2022 09:33:17 +1000
Lines: 18
Message-ID: <jgfe9tFpcloU1@mid.individual.net>
References: <b067e4e7-bb62-4130-950c-5663f4983806n@googlegroups.com>
<jelhoiF4duoU3@mid.individual.net>
<d90a77db-a37e-4619-94b2-a83d49e87f92n@googlegroups.com>
Mime-Version: 1.0
Content-Type: text/plain; charset=UTF-8; format=flowed
Content-Transfer-Encoding: 7bit
X-Trace: individual.net FKVZN4y91i5zolZGU8mKhw5TbKj3DSpbC2aYjInwekh+tUul5t
Cancel-Lock: sha1:MxhGKdNW2jkEdd4EOtCNMNg07Eo=
User-Agent: Mozilla/5.0 (Windows NT 10.0; Win64; x64; rv:91.0) Gecko/20100101
Thunderbird/91.10.0
Content-Language: en-GB
In-Reply-To: <d90a77db-a37e-4619-94b2-a83d49e87f92n@googlegroups.com>
 by: Sylvia Else - Thu, 9 Jun 2022 23:33 UTC

On 08-June-22 6:42 pm, Tom Capizzi wrote:
> On Wednesday, May 18, 2022 at 8:36:38 PM UTC-4, Sylvia Else wrote:
>> On 16-May-22 11:48 am, Tom Capizzi wrote:
>>
>> <snipped due to binary content, apparently>
>>
>> Trying to redefine measurement is a standard technique for relativity
>> deniers. Yet relativity is answering the question "what result will I
>> get if I measure using my usual tools?" And it does that very well.
>>
>> Sylvia.
>
> I question the validity of any comment that assumes a pure text message is "binary content".

My news server rejected my reply based on alleged binary content. It
accepted it after I snipped the text. What more can I say?

Sylvia.

Re: Special Relativity Fatal Error

<6e7d0f54-ee55-450e-9106-f6428f76738en@googlegroups.com>

  copy mid

https://www.novabbs.com/tech/article-flat.php?id=91595&group=sci.physics.relativity#91595

  copy link   Newsgroups: sci.physics.relativity
X-Received: by 2002:ac8:7e84:0:b0:304:f1fd:8f3a with SMTP id w4-20020ac87e84000000b00304f1fd8f3amr17062866qtj.670.1654820871951;
Thu, 09 Jun 2022 17:27:51 -0700 (PDT)
X-Received: by 2002:a05:6214:29ed:b0:46b:937f:6140 with SMTP id
jv13-20020a05621429ed00b0046b937f6140mr19395768qvb.129.1654820871791; Thu, 09
Jun 2022 17:27:51 -0700 (PDT)
Path: i2pn2.org!i2pn.org!usenet.blueworldhosting.com!feed1.usenet.blueworldhosting.com!peer01.iad!feed-me.highwinds-media.com!news.highwinds-media.com!news-out.google.com!nntp.google.com!postnews.google.com!google-groups.googlegroups.com!not-for-mail
Newsgroups: sci.physics.relativity
Date: Thu, 9 Jun 2022 17:27:51 -0700 (PDT)
In-Reply-To: <jgfe9tFpcloU1@mid.individual.net>
Injection-Info: google-groups.googlegroups.com; posting-host=209.6.152.121; posting-account=anpm0goAAAD7eq4-R7Tlsnov4nyr6Xqb
NNTP-Posting-Host: 209.6.152.121
References: <b067e4e7-bb62-4130-950c-5663f4983806n@googlegroups.com>
<jelhoiF4duoU3@mid.individual.net> <d90a77db-a37e-4619-94b2-a83d49e87f92n@googlegroups.com>
<jgfe9tFpcloU1@mid.individual.net>
User-Agent: G2/1.0
MIME-Version: 1.0
Message-ID: <6e7d0f54-ee55-450e-9106-f6428f76738en@googlegroups.com>
Subject: Re: Special Relativity Fatal Error
From: tgcapi...@gmail.com (Tom Capizzi)
Injection-Date: Fri, 10 Jun 2022 00:27:51 +0000
Content-Type: text/plain; charset="UTF-8"
X-Received-Bytes: 2194
 by: Tom Capizzi - Fri, 10 Jun 2022 00:27 UTC

On Thursday, June 9, 2022 at 7:33:20 PM UTC-4, Sylvia Else wrote:
> On 08-June-22 6:42 pm, Tom Capizzi wrote:
> > On Wednesday, May 18, 2022 at 8:36:38 PM UTC-4, Sylvia Else wrote:
> >> On 16-May-22 11:48 am, Tom Capizzi wrote:
> >>
> >> <snipped due to binary content, apparently>
> >>
> >> Trying to redefine measurement is a standard technique for relativity
> >> deniers. Yet relativity is answering the question "what result will I
> >> get if I measure using my usual tools?" And it does that very well.
> >>
> >> Sylvia.
> >
> > I question the validity of any comment that assumes a pure text message is "binary content".
> My news server rejected my reply based on alleged binary content. It
> accepted it after I snipped the text. What more can I say?
>
> Sylvia.

But it did not snip the content when it displayed the message to you? If so, I can reformat the paragraphs so it looks less like binary data.

Re: Special Relativity Fatal Error

<a401249d-7579-4bd9-b7c0-c05e5ea96943n@googlegroups.com>

  copy mid

https://www.novabbs.com/tech/article-flat.php?id=91596&group=sci.physics.relativity#91596

  copy link   Newsgroups: sci.physics.relativity
X-Received: by 2002:a05:6214:e4f:b0:46b:b33f:6e65 with SMTP id o15-20020a0562140e4f00b0046bb33f6e65mr14746005qvc.50.1654822179635;
Thu, 09 Jun 2022 17:49:39 -0700 (PDT)
X-Received: by 2002:a05:620a:372c:b0:6a7:3ad2:68cd with SMTP id
de44-20020a05620a372c00b006a73ad268cdmr3046329qkb.418.1654822179467; Thu, 09
Jun 2022 17:49:39 -0700 (PDT)
Path: i2pn2.org!i2pn.org!weretis.net!feeder8.news.weretis.net!proxad.net!feeder1-2.proxad.net!209.85.160.216.MISMATCH!news-out.google.com!nntp.google.com!postnews.google.com!google-groups.googlegroups.com!not-for-mail
Newsgroups: sci.physics.relativity
Date: Thu, 9 Jun 2022 17:49:39 -0700 (PDT)
In-Reply-To: <1ccdd138-0756-49d9-b980-039996d3392fn@googlegroups.com>
Injection-Info: google-groups.googlegroups.com; posting-host=2601:601:1700:7df0:e06d:7e32:7d75:63b7;
posting-account=mPYpNwoAAADYT6u25jo4wRqpXbzZAAhf
NNTP-Posting-Host: 2601:601:1700:7df0:e06d:7e32:7d75:63b7
References: <b067e4e7-bb62-4130-950c-5663f4983806n@googlegroups.com>
<5086313b-2d82-4e27-9aae-e1dd2cf7a954n@googlegroups.com> <60df787e-d27b-4f10-b7b2-03b3692ee97cn@googlegroups.com>
<_bmdnXI-c7KTIT3_nZ2dnUU7_8zNnZ2d@giganews.com> <1ccdd138-0756-49d9-b980-039996d3392fn@googlegroups.com>
User-Agent: G2/1.0
MIME-Version: 1.0
Message-ID: <a401249d-7579-4bd9-b7c0-c05e5ea96943n@googlegroups.com>
Subject: Re: Special Relativity Fatal Error
From: fultonis...@gmail.com (Stan Fultoni)
Injection-Date: Fri, 10 Jun 2022 00:49:39 +0000
Content-Type: text/plain; charset="UTF-8"
Content-Transfer-Encoding: quoted-printable
 by: Stan Fultoni - Fri, 10 Jun 2022 00:49 UTC

On Thursday, June 9, 2022 at 1:09:30 PM UTC-7, tgca...@gmail.com wrote:
> Albert Einstein defined a legitimate indirect measurement as one that produces the
> same value as a direct measurement would.

He was simply reciting the well-known definition of a standard inertial coordinate system, i.e., an orthogonal grid of standard rulers, to which he later added clocks, all mutually at rest and inertially synchronized. By this operational process we can establish a system x,t of inertial coordinates (in terms of which the equations of physics take their simple homogeneous and isotropic form). Likewise we can carry out the same construction with rulers and clocks all mutually at rest in some other frame, moving at speed v relative to the first, to establish the system x',t' of inertial coordinates. The entire content of special relativity can then be expressed by the fact that those systems are related by x'=(x-vt)g, t'=(t-vx)g where g=1/sqrt(1-v^2). This is an inevitable consequence of the empirical fact that every quantity of energy E has inertia E/c^2.

> ... my argument stands. And since it actually leads to the same predictions of
> measurement as the Einstein Interpretation...

Excellent. So you agree with the empirical fact that standard inertial coordinate systems (defined as above) are related by Lorentz transformations. All the consequence of special relativity follow from this, so it appears you are in complete agreement with special relativity. The only mystery is why you post a message that seems intended to convey some kind of profound disagreement with special relativity, when in fact you actually agree with it completely.

Re: Special Relativity Fatal Error

<9bee2b21-b095-4dd0-a38d-dee29f1b4d5fn@googlegroups.com>

  copy mid

https://www.novabbs.com/tech/article-flat.php?id=91598&group=sci.physics.relativity#91598

  copy link   Newsgroups: sci.physics.relativity
X-Received: by 2002:a05:6214:c26:b0:464:3fdd:a3e6 with SMTP id a6-20020a0562140c2600b004643fdda3e6mr49854567qvd.113.1654825139508;
Thu, 09 Jun 2022 18:38:59 -0700 (PDT)
X-Received: by 2002:ad4:5de1:0:b0:465:5361:1e0 with SMTP id
jn1-20020ad45de1000000b00465536101e0mr34601618qvb.19.1654825139289; Thu, 09
Jun 2022 18:38:59 -0700 (PDT)
Path: i2pn2.org!i2pn.org!weretis.net!feeder8.news.weretis.net!proxad.net!feeder1-2.proxad.net!209.85.160.216.MISMATCH!news-out.google.com!nntp.google.com!postnews.google.com!google-groups.googlegroups.com!not-for-mail
Newsgroups: sci.physics.relativity
Date: Thu, 9 Jun 2022 18:38:58 -0700 (PDT)
In-Reply-To: <n2t4ah5pr2ri9g1jj11gq4fnehbkcac55a@4ax.com>
Injection-Info: google-groups.googlegroups.com; posting-host=209.6.152.121; posting-account=anpm0goAAAD7eq4-R7Tlsnov4nyr6Xqb
NNTP-Posting-Host: 209.6.152.121
References: <b067e4e7-bb62-4130-950c-5663f4983806n@googlegroups.com>
<5086313b-2d82-4e27-9aae-e1dd2cf7a954n@googlegroups.com> <60df787e-d27b-4f10-b7b2-03b3692ee97cn@googlegroups.com>
<_bmdnXI-c7KTIT3_nZ2dnUU7_8zNnZ2d@giganews.com> <1ccdd138-0756-49d9-b980-039996d3392fn@googlegroups.com>
<n2t4ah5pr2ri9g1jj11gq4fnehbkcac55a@4ax.com>
User-Agent: G2/1.0
MIME-Version: 1.0
Message-ID: <9bee2b21-b095-4dd0-a38d-dee29f1b4d5fn@googlegroups.com>
Subject: Re: Special Relativity Fatal Error
From: tgcapi...@gmail.com (Tom Capizzi)
Injection-Date: Fri, 10 Jun 2022 01:38:59 +0000
Content-Type: text/plain; charset="UTF-8"
Content-Transfer-Encoding: quoted-printable
 by: Tom Capizzi - Fri, 10 Jun 2022 01:38 UTC

On Thursday, June 9, 2022 at 6:35:09 PM UTC-4, Ricardo Jimenez wrote:
> On Thu, 9 Jun 2022 13:09:28 -0700 (PDT), Tom Capizzi
> <tgca...@gmail.com> wrote:
>
> >So, I submit the correction. Now we can argue, and convince. The correct protocol for measurement is the Euclidean operation to measure parallelness, the dot product. In its algebraic form, it is routinely used to project components that are parallel and perpendicular to a velocity vector, since relativistic effects only occur in the direction parallel to motion. Normal components are invariant.
> Are you assuming that the usual Pythagorean theorem is valid in the
> (x,t) plane rather than the hyperbolic one?

That's actually a tricky question. In the first place, it depends on whether you're talking about Euclidean geometry or Minkowski geometry. The big picture of my argument is that special relativity is based on a lie, and without the lie, geometry rules apply. In other words, special relativity is a corrupted version of hyperbolic trigonometry. All that being said, relativity is all mathematics. It becomes physics when you install a distribution of particles that move, but basic relativity is all about abstract mathematics. Any attempt to inject physics into this math corrupts it. It is whole and in total agreement with all the existing experimental data as is. So, if there is no special relativity, do we need Minkowski? The reason I ask is that Euclidean vectors are real, and in general have the Pythagorean sum. But the universe is complex, and the imaginary part is directly proportional to Proper velocity. If we add the squares of the real and imaginary contributions, the result is the relativistic invariant: (ct)² + (ir)² = ct²- r² = s². So, is s² a Pythagorean sum of the squares of complex numbers or is it a hyperbolic invariant? Does it make a difference?

But fundamental concepts, like "real" and "perpendicular" do not mean the same thing in Euclidean and Minkowski geometries. So, instead of Pythagorean or hyperbolic, I use symmetry operations. The simplest pair is addition and subtraction. Given 2 positive definite quantities, I define two combinations, Σ = ct+r and Δ = ct-r. Or in terms of quadratic measures, (ct)² + (r)² = S² and (ct)² - (r)² = s².. S² is not invariant with respect to relative velocity, so all observers measure different values. But, it is still the square of the hypotenuse of a Pythagorean triangle. One in particular dominates all the geometry. It is customary to assign any observer an initial position of (0,0,0,0) with an initial velocity of (c,0,0,0). So, we select an origin and a unit step, and put a baseline through them. At the unit step, we construct a perpendicular. This is the Proper velocity axis. We can locate a point anywhere on this axis, and it will represent our observer's inertial frame after a boost. The Pythagorean sum of squares is (c)² + (γv)² = (c)² + (γβc)² = (1+γ²β²)c² = γ²c² = (γc)². This is an interesting result, because (γc,γv) is the relativistic 4-velocity, the invariant of which is (γc)²-(γv)² = (c)². The point, (γc,γv), is on a hyperbola, and the semi-major axis of the hyperbola is c, vertex is (c,0,0,0). There is an isomorphism at the heart of hyperbolic trigonometry that maps hyperbolic angles to circular angles such that for every pair (w,tilt) related this way, cosh(w) = sec(tilt), sinh(w) = tan(tilt) and tanh(w) = sin(tilt). This is the fundamental equation of special relativity. A small change in rapidity results in an even smaller change in tilt angle such that: dw/dtilt = γ = sec(tilt). In other words, dtilt = dw*cos(tilt), the change in tilt angle depends on the current value of tilt. As tilt gets larger, cos(tilt) gets smaller. And this explains the invariance of c. It is the limit of the cosine projections of Proper velocity as rapidity and Proper velocity go to infinity. Lightspeed is the cosine projection of infinite Proper velocity. It is the same everywhere in vacuum because infinity is the same everywhere.

So, I guess the answer to your question is yes to Pythagoras, and hyperbolic trigonometry is isomorphic.

Re: Special Relativity Fatal Error

<2948cbc7-bc6b-49bb-ba34-b094243fece5n@googlegroups.com>

  copy mid

https://www.novabbs.com/tech/article-flat.php?id=91600&group=sci.physics.relativity#91600

  copy link   Newsgroups: sci.physics.relativity
X-Received: by 2002:ad4:596b:0:b0:46b:cc90:5a87 with SMTP id eq11-20020ad4596b000000b0046bcc905a87mr9556791qvb.59.1654825804117;
Thu, 09 Jun 2022 18:50:04 -0700 (PDT)
X-Received: by 2002:a05:6214:5298:b0:469:dbf0:899a with SMTP id
kj24-20020a056214529800b00469dbf0899amr26157619qvb.103.1654825803883; Thu, 09
Jun 2022 18:50:03 -0700 (PDT)
Path: i2pn2.org!i2pn.org!usenet.blueworldhosting.com!feed1.usenet.blueworldhosting.com!peer03.iad!feed-me.highwinds-media.com!news.highwinds-media.com!news-out.google.com!nntp.google.com!postnews.google.com!google-groups.googlegroups.com!not-for-mail
Newsgroups: sci.physics.relativity
Date: Thu, 9 Jun 2022 18:50:03 -0700 (PDT)
In-Reply-To: <a401249d-7579-4bd9-b7c0-c05e5ea96943n@googlegroups.com>
Injection-Info: google-groups.googlegroups.com; posting-host=209.6.152.121; posting-account=anpm0goAAAD7eq4-R7Tlsnov4nyr6Xqb
NNTP-Posting-Host: 209.6.152.121
References: <b067e4e7-bb62-4130-950c-5663f4983806n@googlegroups.com>
<5086313b-2d82-4e27-9aae-e1dd2cf7a954n@googlegroups.com> <60df787e-d27b-4f10-b7b2-03b3692ee97cn@googlegroups.com>
<_bmdnXI-c7KTIT3_nZ2dnUU7_8zNnZ2d@giganews.com> <1ccdd138-0756-49d9-b980-039996d3392fn@googlegroups.com>
<a401249d-7579-4bd9-b7c0-c05e5ea96943n@googlegroups.com>
User-Agent: G2/1.0
MIME-Version: 1.0
Message-ID: <2948cbc7-bc6b-49bb-ba34-b094243fece5n@googlegroups.com>
Subject: Re: Special Relativity Fatal Error
From: tgcapi...@gmail.com (Tom Capizzi)
Injection-Date: Fri, 10 Jun 2022 01:50:04 +0000
Content-Type: text/plain; charset="UTF-8"
Content-Transfer-Encoding: quoted-printable
X-Received-Bytes: 4224
 by: Tom Capizzi - Fri, 10 Jun 2022 01:50 UTC

On Thursday, June 9, 2022 at 8:49:41 PM UTC-4, Stan Fultoni wrote:
> On Thursday, June 9, 2022 at 1:09:30 PM UTC-7, tgca...@gmail.com wrote:
> > Albert Einstein defined a legitimate indirect measurement as one that produces the
> > same value as a direct measurement would.
> He was simply reciting the well-known definition of a standard inertial coordinate system, i.e., an orthogonal grid of standard rulers, to which he later added clocks, all mutually at rest and inertially synchronized. By this operational process we can establish a system x,t of inertial coordinates (in terms of which the equations of physics take their simple homogeneous and isotropic form). Likewise we can carry out the same construction with rulers and clocks all mutually at rest in some other frame, moving at speed v relative to the first, to establish the system x',t' of inertial coordinates. The entire content of special relativity can then be expressed by the fact that those systems are related by x'=(x-vt)g, t'=(t-vx)g where g=1/sqrt(1-v^2). This is an inevitable consequence of the empirical fact that every quantity of energy E has inertia E/c^2.
>
> > ... my argument stands. And since it actually leads to the same predictions of
> > measurement as the Einstein Interpretation...
>
> Excellent. So you agree with the empirical fact that standard inertial coordinate systems (defined as above) are related by Lorentz transformations. All the consequence of special relativity follow from this, so it appears you are in complete agreement with special relativity. The only mystery is why you post a message that seems intended to convey some kind of profound disagreement with special relativity, when in fact you actually agree with it completely.

Silly boy. I agree with reality that measures shrunken vectors of time and space. I do not agree that physics has anything to do with it. It is purely mathematical. Special relativity is a corruption of the mathematics. As far as Lorentz is concerned, the elements of every Lorentz matrix are the coordinates of a point on the unit hyperbola. The Lorentz transform is nothing more than a displacement of a point on the unit hyperbola to another point on the unit hyperbola. Since you apparently just skimmed my post, you missed the part where I explain that NOTHING EVER SHRINKS. It's a geometric illusion like an oasis, or gravity lensing. In those, the things we see are real, but they are not located where they appear to be. Relativity is the real projection of an invariant complex magnitude. How you could assert "when in fact you actually agree with it completely" is beyond me.

Re: Special Relativity Fatal Error

<jgfmmkFqlk7U1@mid.individual.net>

  copy mid

https://www.novabbs.com/tech/article-flat.php?id=91601&group=sci.physics.relativity#91601

  copy link   Newsgroups: sci.physics.relativity
Path: i2pn2.org!i2pn.org!weretis.net!feeder8.news.weretis.net!3.eu.feeder.erje.net!feeder.erje.net!fu-berlin.de!uni-berlin.de!individual.net!not-for-mail
From: syl...@email.invalid (Sylvia Else)
Newsgroups: sci.physics.relativity
Subject: Re: Special Relativity Fatal Error
Date: Fri, 10 Jun 2022 11:56:34 +1000
Lines: 35
Message-ID: <jgfmmkFqlk7U1@mid.individual.net>
References: <b067e4e7-bb62-4130-950c-5663f4983806n@googlegroups.com>
<jelhoiF4duoU3@mid.individual.net>
<d90a77db-a37e-4619-94b2-a83d49e87f92n@googlegroups.com>
<jgfe9tFpcloU1@mid.individual.net>
<6e7d0f54-ee55-450e-9106-f6428f76738en@googlegroups.com>
Mime-Version: 1.0
Content-Type: text/plain; charset=UTF-8; format=flowed
Content-Transfer-Encoding: 7bit
X-Trace: individual.net 6N4JcV9D3cG3tZ+dJ0WxXAHX5+3e+TXD2rRCvHw3DLoBUBRWU6
Cancel-Lock: sha1:WL0YB0jSodyrQN6N0Mp5sjBakBc=
User-Agent: Mozilla/5.0 (Windows NT 10.0; Win64; x64; rv:91.0) Gecko/20100101
Thunderbird/91.10.0
Content-Language: en-GB
In-Reply-To: <6e7d0f54-ee55-450e-9106-f6428f76738en@googlegroups.com>
 by: Sylvia Else - Fri, 10 Jun 2022 01:56 UTC

On 10-June-22 10:27 am, Tom Capizzi wrote:
> On Thursday, June 9, 2022 at 7:33:20 PM UTC-4, Sylvia Else wrote:
>> On 08-June-22 6:42 pm, Tom Capizzi wrote:
>>> On Wednesday, May 18, 2022 at 8:36:38 PM UTC-4, Sylvia Else wrote:
>>>> On 16-May-22 11:48 am, Tom Capizzi wrote:
>>>>
>>>> <snipped due to binary content, apparently>
>>>>
>>>> Trying to redefine measurement is a standard technique for relativity
>>>> deniers. Yet relativity is answering the question "what result will I
>>>> get if I measure using my usual tools?" And it does that very well.
>>>>
>>>> Sylvia.
>>>
>>> I question the validity of any comment that assumes a pure text message is "binary content".
>> My news server rejected my reply based on alleged binary content. It
>> accepted it after I snipped the text. What more can I say?
>>
>> Sylvia.
>
> But it did not snip the content when it displayed the message to you? If so, I can reformat the paragraphs so it looks less like binary data.

It appears to object somewhere within the sentence "Frames whose
relative velocity was so small that the relativistic correction factor
was unity." Snipping that and all that follows posts OK to alt.test.
Leave it in and snip the rest, and it chokes.

It also objects if I snip that sentence and all that follows, and just
type random characters at the end of your paragraph.

So I think it's a length issue. Most people do not write single
paragraphs of that length, so I'm not surprised I've not encountered it
before.

Sylvia.

Re: Special Relativity Fatal Error

<aaa835de-947e-4b1a-a20d-f2522cffead7n@googlegroups.com>

  copy mid

https://www.novabbs.com/tech/article-flat.php?id=91603&group=sci.physics.relativity#91603

  copy link   Newsgroups: sci.physics.relativity
X-Received: by 2002:a05:620a:2950:b0:6a3:a838:ad70 with SMTP id n16-20020a05620a295000b006a3a838ad70mr29772360qkp.14.1654829439840;
Thu, 09 Jun 2022 19:50:39 -0700 (PDT)
X-Received: by 2002:a37:f609:0:b0:6a6:a6f7:3ead with SMTP id
y9-20020a37f609000000b006a6a6f73eadmr23007443qkj.501.1654829439675; Thu, 09
Jun 2022 19:50:39 -0700 (PDT)
Path: i2pn2.org!i2pn.org!usenet.blueworldhosting.com!feed1.usenet.blueworldhosting.com!peer03.iad!feed-me.highwinds-media.com!news.highwinds-media.com!news-out.google.com!nntp.google.com!postnews.google.com!google-groups.googlegroups.com!not-for-mail
Newsgroups: sci.physics.relativity
Date: Thu, 9 Jun 2022 19:50:39 -0700 (PDT)
In-Reply-To: <2948cbc7-bc6b-49bb-ba34-b094243fece5n@googlegroups.com>
Injection-Info: google-groups.googlegroups.com; posting-host=2601:601:1700:7df0:e06d:7e32:7d75:63b7;
posting-account=mPYpNwoAAADYT6u25jo4wRqpXbzZAAhf
NNTP-Posting-Host: 2601:601:1700:7df0:e06d:7e32:7d75:63b7
References: <b067e4e7-bb62-4130-950c-5663f4983806n@googlegroups.com>
<5086313b-2d82-4e27-9aae-e1dd2cf7a954n@googlegroups.com> <60df787e-d27b-4f10-b7b2-03b3692ee97cn@googlegroups.com>
<_bmdnXI-c7KTIT3_nZ2dnUU7_8zNnZ2d@giganews.com> <1ccdd138-0756-49d9-b980-039996d3392fn@googlegroups.com>
<a401249d-7579-4bd9-b7c0-c05e5ea96943n@googlegroups.com> <2948cbc7-bc6b-49bb-ba34-b094243fece5n@googlegroups.com>
User-Agent: G2/1.0
MIME-Version: 1.0
Message-ID: <aaa835de-947e-4b1a-a20d-f2522cffead7n@googlegroups.com>
Subject: Re: Special Relativity Fatal Error
From: fultonis...@gmail.com (Stan Fultoni)
Injection-Date: Fri, 10 Jun 2022 02:50:39 +0000
Content-Type: text/plain; charset="UTF-8"
Content-Transfer-Encoding: quoted-printable
X-Received-Bytes: 3534
 by: Stan Fultoni - Fri, 10 Jun 2022 02:50 UTC

On Thursday, June 9, 2022 at 6:50:05 PM UTC-7, tgca...@gmail.com wrote:
> > > ... my argument ... actually leads to the same predictions of measurement
> > > as the Einstein Interpretation...
> >
> > Excellent. So you agree with the empirical fact that standard inertial coordinate systems (defined as above) are related by Lorentz transformations. All the consequence of special relativity follow from this, so it appears you are in complete agreement with special relativity. The only mystery is why you post a message that seems intended to convey some kind of profound disagreement with special relativity, when in fact you actually agree with it completely.
>
> I do not agree that physics has anything to do with it. It is purely mathematical.

Passive transformations are purely mathematical, but active transformations are not. It is the active transformations that give Lorentz invariance it's physical significance. Again, since you assure me that your argument leads to the same predictions of measurements as special relativity, you agree that all the laws of physics are Lorentz invariant, meaning that inertia-based coordinate systems are related by Lorentz transformations. Excellent..

> Relativity is the real projection of an invariant complex magnitude.

Again, you are talking only about passive transformations, which are pure math. Physics is about active transformations that reveal the Lorentz invariance of all physical laws.

> How you could assert "when in fact you actually agree with it completely" is beyond me.

That's may be true, i.e., it may be beyond you. But based on your assurance that you agree with all the empirical manifestations of Lorentz invariance, you agree completely with special relativity. If you are determined not to agree, you need to retract your claim -- which then exposes you to empirical falsification.

Re: Special Relativity Fatal Error

<f036eed3-7528-4a31-b594-f77315a05237n@googlegroups.com>

  copy mid

https://www.novabbs.com/tech/article-flat.php?id=91607&group=sci.physics.relativity#91607

  copy link   Newsgroups: sci.physics.relativity
X-Received: by 2002:a05:6214:c26:b0:464:3fdd:a3e6 with SMTP id a6-20020a0562140c2600b004643fdda3e6mr50155267qvd.113.1654835151898;
Thu, 09 Jun 2022 21:25:51 -0700 (PDT)
X-Received: by 2002:a05:6214:5005:b0:461:c843:98e7 with SMTP id
jo5-20020a056214500500b00461c84398e7mr32412755qvb.16.1654835151717; Thu, 09
Jun 2022 21:25:51 -0700 (PDT)
Path: i2pn2.org!i2pn.org!usenet.blueworldhosting.com!feed1.usenet.blueworldhosting.com!peer03.iad!feed-me.highwinds-media.com!news.highwinds-media.com!news-out.google.com!nntp.google.com!postnews.google.com!google-groups.googlegroups.com!not-for-mail
Newsgroups: sci.physics.relativity
Date: Thu, 9 Jun 2022 21:25:51 -0700 (PDT)
In-Reply-To: <aaa835de-947e-4b1a-a20d-f2522cffead7n@googlegroups.com>
Injection-Info: google-groups.googlegroups.com; posting-host=209.6.152.121; posting-account=anpm0goAAAD7eq4-R7Tlsnov4nyr6Xqb
NNTP-Posting-Host: 209.6.152.121
References: <b067e4e7-bb62-4130-950c-5663f4983806n@googlegroups.com>
<5086313b-2d82-4e27-9aae-e1dd2cf7a954n@googlegroups.com> <60df787e-d27b-4f10-b7b2-03b3692ee97cn@googlegroups.com>
<_bmdnXI-c7KTIT3_nZ2dnUU7_8zNnZ2d@giganews.com> <1ccdd138-0756-49d9-b980-039996d3392fn@googlegroups.com>
<a401249d-7579-4bd9-b7c0-c05e5ea96943n@googlegroups.com> <2948cbc7-bc6b-49bb-ba34-b094243fece5n@googlegroups.com>
<aaa835de-947e-4b1a-a20d-f2522cffead7n@googlegroups.com>
User-Agent: G2/1.0
MIME-Version: 1.0
Message-ID: <f036eed3-7528-4a31-b594-f77315a05237n@googlegroups.com>
Subject: Re: Special Relativity Fatal Error
From: tgcapi...@gmail.com (Tom Capizzi)
Injection-Date: Fri, 10 Jun 2022 04:25:51 +0000
Content-Type: text/plain; charset="UTF-8"
Content-Transfer-Encoding: quoted-printable
X-Received-Bytes: 4536
 by: Tom Capizzi - Fri, 10 Jun 2022 04:25 UTC

On Thursday, June 9, 2022 at 10:50:41 PM UTC-4, Stan Fultoni wrote:
> On Thursday, June 9, 2022 at 6:50:05 PM UTC-7, tgca...@gmail.com wrote:
> > > > ... my argument ... actually leads to the same predictions of measurement
> > > > as the Einstein Interpretation...
> > >
> > > Excellent. So you agree with the empirical fact that standard inertial coordinate systems (defined as above) are related by Lorentz transformations. All the consequence of special relativity follow from this, so it appears you are in complete agreement with special relativity. The only mystery is why you post a message that seems intended to convey some kind of profound disagreement with special relativity, when in fact you actually agree with it completely.
> >
> > I do not agree that physics has anything to do with it. It is purely mathematical.
> Passive transformations are purely mathematical, but active transformations are not. It is the active transformations that give Lorentz invariance it's physical significance. Again, since you assure me that your argument leads to the same predictions of measurements as special relativity, you agree that all the laws of physics are Lorentz invariant, meaning that inertia-based coordinate systems are related by Lorentz transformations. Excellent.
> > Relativity is the real projection of an invariant complex magnitude.
> Again, you are talking only about passive transformations, which are pure math. Physics is about active transformations that reveal the Lorentz invariance of all physical laws.
> > How you could assert "when in fact you actually agree with it completely" is beyond me.
> That's may be true, i.e., it may be beyond you. But based on your assurance that you agree with all the empirical manifestations of Lorentz invariance, you agree completely with special relativity. If you are determined not to agree, you need to retract your claim -- which then exposes you to empirical falsification.

Fine. Argue with your strawmen. As far as I'm concerned special relativity is junk science. I agree that the results of measurements agree with the predictions of the Lorentz transform. However, I reject the claim that anything physically shrinks. It's a hyperbolic rotation, w. By definition, hyperbolic magnitude is invariant with respect to relative velocity, c tanh(w). Nothing shrinks.

As to your "argument", I don't accept your classification of active and passive transformations, nor do I accept the conclusions you assert without proof. Embedded in the concept of rigid rotation into Elsewhere is the implication of the existence of dimensions outside the Minkowski 4. When special relativity restores objective reality and includes 4 complex coordinates instead of real scalars, then we can talk about how much I agree with it. As it stands, special relativity is a fraud.

Re: Special Relativity Fatal Error

<e541181c-65a1-4d61-9bcf-4dcc77b482f8n@googlegroups.com>

  copy mid

https://www.novabbs.com/tech/article-flat.php?id=91608&group=sci.physics.relativity#91608

  copy link   Newsgroups: sci.physics.relativity
X-Received: by 2002:a05:622a:120e:b0:305:78:36ba with SMTP id y14-20020a05622a120e00b00305007836bamr10664381qtx.386.1654837818041;
Thu, 09 Jun 2022 22:10:18 -0700 (PDT)
X-Received: by 2002:a05:622a:11ca:b0:304:c340:c37f with SMTP id
n10-20020a05622a11ca00b00304c340c37fmr34604047qtk.336.1654837817248; Thu, 09
Jun 2022 22:10:17 -0700 (PDT)
Path: i2pn2.org!i2pn.org!weretis.net!feeder8.news.weretis.net!proxad.net!feeder1-2.proxad.net!209.85.160.216.MISMATCH!news-out.google.com!nntp.google.com!postnews.google.com!google-groups.googlegroups.com!not-for-mail
Newsgroups: sci.physics.relativity
Date: Thu, 9 Jun 2022 22:10:17 -0700 (PDT)
In-Reply-To: <f036eed3-7528-4a31-b594-f77315a05237n@googlegroups.com>
Injection-Info: google-groups.googlegroups.com; posting-host=2601:601:1700:7df0:e06d:7e32:7d75:63b7;
posting-account=mPYpNwoAAADYT6u25jo4wRqpXbzZAAhf
NNTP-Posting-Host: 2601:601:1700:7df0:e06d:7e32:7d75:63b7
References: <b067e4e7-bb62-4130-950c-5663f4983806n@googlegroups.com>
<5086313b-2d82-4e27-9aae-e1dd2cf7a954n@googlegroups.com> <60df787e-d27b-4f10-b7b2-03b3692ee97cn@googlegroups.com>
<_bmdnXI-c7KTIT3_nZ2dnUU7_8zNnZ2d@giganews.com> <1ccdd138-0756-49d9-b980-039996d3392fn@googlegroups.com>
<a401249d-7579-4bd9-b7c0-c05e5ea96943n@googlegroups.com> <2948cbc7-bc6b-49bb-ba34-b094243fece5n@googlegroups.com>
<aaa835de-947e-4b1a-a20d-f2522cffead7n@googlegroups.com> <f036eed3-7528-4a31-b594-f77315a05237n@googlegroups.com>
User-Agent: G2/1.0
MIME-Version: 1.0
Message-ID: <e541181c-65a1-4d61-9bcf-4dcc77b482f8n@googlegroups.com>
Subject: Re: Special Relativity Fatal Error
From: fultonis...@gmail.com (Stan Fultoni)
Injection-Date: Fri, 10 Jun 2022 05:10:18 +0000
Content-Type: text/plain; charset="UTF-8"
Content-Transfer-Encoding: quoted-printable
 by: Stan Fultoni - Fri, 10 Jun 2022 05:10 UTC

On Thursday, June 9, 2022 at 9:25:53 PM UTC-7, tgca...@gmail.com wrote:
> > > > > ... my argument ... actually leads to the same predictions of measurement
> > > > > as the Einstein Interpretation...
> > > >
> > > > Excellent. So you agree with the empirical fact that standard inertial coordinate systems (defined as above) are related by Lorentz transformations. All the consequence of special relativity follow from this, so it appears you are in complete agreement with special relativity. The only mystery is why you post a message that seems intended to convey some kind of profound disagreement with special relativity, when in fact you actually agree with it completely.
> > >
> > > I do not agree that physics has anything to do with it. It is purely mathematical.
> > Passive transformations are purely mathematical, but active transformations are not. It is the active transformations that give Lorentz invariance it's physical significance. Again, since you assure me that your argument leads to the same predictions of measurements as special relativity, you agree that all the laws of physics are Lorentz invariant, meaning that inertia-based coordinate systems are related by Lorentz transformations. Excellent.
> > > Relativity is the real projection of an invariant complex magnitude.
> > Again, you are talking only about passive transformations, which are pure math. Physics is about active transformations that reveal the Lorentz invariance of all physical laws.
> > > How you could assert "when in fact you actually agree with it completely" is beyond me.
> > That's may be true, i.e., it may be beyond you. But based on your assurance that you agree with all the empirical manifestations of Lorentz invariance, you agree completely with special relativity. If you are determined not to agree, you need to retract your claim -- which then exposes you to empirical falsification.
>
> I agree that the results of measurements agree with the predictions of the Lorentz transform.

Presumably you mean transformation, not transform. And that's great, because it signifies that you agree that the laws of physics are (locally) Lorentz invariant, and that inertia-based coordinate systems are related by Lorentz transformations, which is the entire content of special relativity.

> However, I reject the claim that anything physically shrinks.

Rather than playing semantic games, let's just be clear about what we mean: If, in terms of inertial coordinate system S, the simultaneous distance between the ends of a solid rod at rest in S is L, and if we then gently accelerate the rod and allow it to reach equilibrium moving at speed v, the simultaneous distance between the ends of the rod (still in terms of S) is now L sqrt(1-v^2/c^2). This is the prediction of special relativity, and you have said you agree with it, so you really have no point.

Remember, every school boy knows about the description in terms of hyperbolic rotation. Poincare and Minkowski emphasized that. This is entirely consistent with special relativity. Also, remember that Maxwell, Heaviside, and Searle already showed in the 1800's that Maxwell's equations imply that any object governed by electromagnetic interactions must shrink in the direction of motion by the factor sqrt(1-v^2/c^2). Maxwell's equations are Lorentz invariant, so this shouldn't be surprising.

> It's a hyperbolic rotation, w. Nothing shrinks.

You're confusing passive and active transformations. An actual active acceleration of a material object is not identical to a simple hyperbolic rotation, it's more complicated, although at equilibrium for sufficiently gentle acceleration it asymptotically approaches it. This is just Relativity 101..

> I don't accept your classification of active and passive transformations....

It isn't a classification, it is simply a statement of well known fact, clarifying an important distinction that often confuses beginners who wonder if relativistic effects are real.

> When special relativity restores objective reality...

Special relativity involves only a single object reality. The notion that relativity is a subjectivist theory with different realities for different "observers" is just a common newbie fallacy. Remember, Minkowski called relativity "The Theory of the Absolute World", and Einstein thought it should be named "Invariant Theory". It was Planck who got people to call it relativity theory.

> and includes 4 complex coordinates instead of real scalars...

Many people (including Einstein) have studied generalizing geometry to include four (or even more) complex dimensions, and of course the Hilbert space of quantum mechanics has complex dimensions (along with the measurement problem), but for the relations of space and time it has not been a fruitful idea. The spatio-temporal relations between events are stubbornly real, as are the eigenvalues of quantum mechanical operators.

> then we can talk about how much I agree with it. As it stands, special relativity is a fraud.

You contradict yourself. You have repeatedly stated you agree with special relativity completely, meaning you agree with all of its empirical predictions, which is all special relativity consists of. But now you say it is a fraud. One of the most important attributes of science is logical consistency. You've got much room for improvement in that department.

Re: Special Relativity Fatal Error

<jgg33dFsidtU1@mid.individual.net>

  copy mid

https://www.novabbs.com/tech/article-flat.php?id=91611&group=sci.physics.relativity#91611

  copy link   Newsgroups: sci.physics.relativity
Path: i2pn2.org!i2pn.org!weretis.net!feeder8.news.weretis.net!news.mixmin.net!news2.arglkargh.de!news.karotte.org!fu-berlin.de!uni-berlin.de!individual.net!not-for-mail
From: ttt_...@web.de (Thomas Heger)
Newsgroups: sci.physics.relativity
Subject: Re: Special Relativity Fatal Error
Date: Fri, 10 Jun 2022 07:28:19 +0200
Lines: 51
Message-ID: <jgg33dFsidtU1@mid.individual.net>
References: <b067e4e7-bb62-4130-950c-5663f4983806n@googlegroups.com> <5086313b-2d82-4e27-9aae-e1dd2cf7a954n@googlegroups.com> <60df787e-d27b-4f10-b7b2-03b3692ee97cn@googlegroups.com> <_bmdnXI-c7KTIT3_nZ2dnUU7_8zNnZ2d@giganews.com>
Mime-Version: 1.0
Content-Type: text/plain; charset=utf-8; format=flowed
Content-Transfer-Encoding: 7bit
X-Trace: individual.net qyE+m4wlyPrr8IK92JkqyQFcIMmfGdQMmnp9uoKwBGjG0BePe0
Cancel-Lock: sha1:E/DnEGsNjC+NfMUP9rq/94+5Oqs=
User-Agent: Mozilla/5.0 (Windows NT 6.0; WOW64; rv:31.0) Gecko/20100101 Thunderbird/31.4.0
In-Reply-To: <_bmdnXI-c7KTIT3_nZ2dnUU7_8zNnZ2d@giganews.com>
 by: Thomas Heger - Fri, 10 Jun 2022 05:28 UTC

Am 08.06.2022 um 17:11 schrieb Tom Roberts:
> On 6/8/22 3:13 AM, Tom Capizzi wrote:
>> On Monday, May 16, 2022 at 4:47:04 AM UTC-4, JanPB wrote:
>>> On Sunday, May 15, 2022 at 6:48:24 PM UTC-7, tgca...@gmail.com
>>> wrote:
>>>> After years of rereading Einstein and arguing with his legions of
>>>> cultists,
>>> Sigh. Yet another crackpot.
>>>
>>> -- Jan
>>
>> Yes. Don't you hate it when crackpot skeptics blather about how it is
>> impossible for there to be anything wrong with special relativity?
>
> Don't you hate it when people don't realize that the mathematics
> underlying Special Relativity has been proven to be as self-consistent
> as is Euclidean geometry, and as is real analysis?

Appartently the universe 'speaks' in a mathematical language, which is
also known as 'geometric algebra'.

For this type of 'university math' we would need complex four vectors,
which are also known as bi-quaternions.

Now it is easy to see, that Einstein didn't use anything similar in his
1905 paper. He didn't even use usual vectors properly.

So how do you connect his 'self-consistent' math with the behaviour of
the universe, if the underlying math is so different?

Einstein mentioned group theory very briefly, but not which group would fit.

For some reason I think, that a certain group called CL_3 would match
the requirements, but cannot find any reference to that particular group
in Einstein's paper.

Therefore, a self-consistent description does not necessarily describe
the real world properly.

To test, whether a mathematical construct would fit to observations,
scientists usually conduct certain tests, known as 'experiments'.

But Einstein shifted real experiments to an imaginary realm, where no
discrepancy between math and tought was possible.

....

TH

Re: Special Relativity Fatal Error

<e38eceea-33ff-467c-acfd-2a555eff781fn@googlegroups.com>

  copy mid

https://www.novabbs.com/tech/article-flat.php?id=91613&group=sci.physics.relativity#91613

  copy link   Newsgroups: sci.physics.relativity
X-Received: by 2002:ae9:f405:0:b0:6a7:1892:8603 with SMTP id y5-20020ae9f405000000b006a718928603mr6725563qkl.293.1654839526401;
Thu, 09 Jun 2022 22:38:46 -0700 (PDT)
X-Received: by 2002:a37:4550:0:b0:6a7:4253:f665 with SMTP id
s77-20020a374550000000b006a74253f665mr1676777qka.171.1654839526286; Thu, 09
Jun 2022 22:38:46 -0700 (PDT)
Path: i2pn2.org!i2pn.org!usenet.blueworldhosting.com!feed1.usenet.blueworldhosting.com!peer03.iad!feed-me.highwinds-media.com!news.highwinds-media.com!news-out.google.com!nntp.google.com!postnews.google.com!google-groups.googlegroups.com!not-for-mail
Newsgroups: sci.physics.relativity
Date: Thu, 9 Jun 2022 22:38:46 -0700 (PDT)
In-Reply-To: <jgg33dFsidtU1@mid.individual.net>
Injection-Info: google-groups.googlegroups.com; posting-host=89.206.14.16; posting-account=I3DWzAoAAACOmZUdDcZ-C0PqAZGVsbW0
NNTP-Posting-Host: 89.206.14.16
References: <b067e4e7-bb62-4130-950c-5663f4983806n@googlegroups.com>
<5086313b-2d82-4e27-9aae-e1dd2cf7a954n@googlegroups.com> <60df787e-d27b-4f10-b7b2-03b3692ee97cn@googlegroups.com>
<_bmdnXI-c7KTIT3_nZ2dnUU7_8zNnZ2d@giganews.com> <jgg33dFsidtU1@mid.individual.net>
User-Agent: G2/1.0
MIME-Version: 1.0
Message-ID: <e38eceea-33ff-467c-acfd-2a555eff781fn@googlegroups.com>
Subject: Re: Special Relativity Fatal Error
From: maluwozn...@gmail.com (Maciej Wozniak)
Injection-Date: Fri, 10 Jun 2022 05:38:46 +0000
Content-Type: text/plain; charset="UTF-8"
X-Received-Bytes: 1636
 by: Maciej Wozniak - Fri, 10 Jun 2022 05:38 UTC

On Friday, 10 June 2022 at 07:28:18 UTC+2, Thomas Heger wrote:

> Appartently the universe 'speaks' in a mathematical language, which is
> also known as 'geometric algebra'.

The universe is silent, like it always was. If you're
applying some mathematical language to it - that's
because you're trained to try.

Re: Special Relativity Fatal Error

<lYCdnQd9Taw_Dz7_nZ2dnUU7_8zNnZ2d@giganews.com>

  copy mid

https://www.novabbs.com/tech/article-flat.php?id=91642&group=sci.physics.relativity#91642

  copy link   Newsgroups: sci.physics.relativity
Path: i2pn2.org!i2pn.org!weretis.net!feeder6.news.weretis.net!1.us.feeder.erje.net!3.us.feeder.erje.net!feeder.erje.net!border1.nntp.dca1.giganews.com!nntp.giganews.com!buffer1.nntp.dca1.giganews.com!news.giganews.com.POSTED!not-for-mail
NNTP-Posting-Date: Fri, 10 Jun 2022 13:51:14 -0500
Date: Fri, 10 Jun 2022 13:51:14 -0500
MIME-Version: 1.0
User-Agent: Mozilla/5.0 (Macintosh; Intel Mac OS X 10.15; rv:91.0)
Gecko/20100101 Thunderbird/91.9.1
Subject: Re: Special Relativity Fatal Error
Content-Language: en-US
Newsgroups: sci.physics.relativity
References: <b067e4e7-bb62-4130-950c-5663f4983806n@googlegroups.com>
<5086313b-2d82-4e27-9aae-e1dd2cf7a954n@googlegroups.com>
<60df787e-d27b-4f10-b7b2-03b3692ee97cn@googlegroups.com>
<_bmdnXI-c7KTIT3_nZ2dnUU7_8zNnZ2d@giganews.com>
<1ccdd138-0756-49d9-b980-039996d3392fn@googlegroups.com>
From: tjrobert...@sbcglobal.net (Tom Roberts)
In-Reply-To: <1ccdd138-0756-49d9-b980-039996d3392fn@googlegroups.com>
Content-Type: text/plain; charset=UTF-8
Content-Transfer-Encoding: 7bit
Message-ID: <lYCdnQd9Taw_Dz7_nZ2dnUU7_8zNnZ2d@giganews.com>
Lines: 35
X-Usenet-Provider: http://www.giganews.com
X-Trace: sv3-xzPczA/CPN3Bfyd6grpLkOAiCTr0gpI2wQFTFPZkpwFtfptLUh45/2T9d1IfwYXbAmWGF9EdUdgnHp4!4j5GYCqQpYrwTiThJzAPF2nMMu0d5wSv3+iRUbeVoFPysvQz3/tWVmelgqpEh0IKQSoYdP1Ikw==
X-Complaints-To: abuse@giganews.com
X-DMCA-Notifications: http://www.giganews.com/info/dmca.html
X-Abuse-and-DMCA-Info: Please be sure to forward a copy of ALL headers
X-Abuse-and-DMCA-Info: Otherwise we will be unable to process your complaint properly
X-Postfilter: 1.3.40
X-Original-Bytes: 2571
 by: Tom Roberts - Fri, 10 Jun 2022 18:51 UTC

[I'll give you a chance to explain your personal vocabulary and describe
your claims in terms other people can understand. If you can't do that,
I'll abandon the thread as useless.]

On 6/9/22 3:09 PM, Tom Capizzi wrote:
> [...] Albert Einstein defined a legitimate indirect measurement as
> one that produces the same value as a direct measurement would.

What do you mean by that?

What is "legitimate" and how did Einstein define it? (That's a word he
never used AFAIK)

What are "direct and indirect measurements"?

What "same value"?

> His own gedanken experiments, and every real experiment show that
> there is no protocol for indirect measurement above some threshold
> velocity that satisfies this requirement of validity.

What is "indirect measurement"?

What "threshold velocity"?

What "requirement of validity"?

> [...]

Your prose is very dense in undefined words -- none of your phrases that
I quoted above are standard, they are your personal vocabulary. Describe
what you mean so others can understand, and we can go on to the rest of
your post.

Tom Roberts

Re: Special Relativity Fatal Error

<65fdf573-e4c2-4974-8bf0-c241fa528606n@googlegroups.com>

  copy mid

https://www.novabbs.com/tech/article-flat.php?id=91645&group=sci.physics.relativity#91645

  copy link   Newsgroups: sci.physics.relativity
X-Received: by 2002:a05:6214:248a:b0:464:5f79:e9ea with SMTP id gi10-20020a056214248a00b004645f79e9eamr43244101qvb.47.1654901733577;
Fri, 10 Jun 2022 15:55:33 -0700 (PDT)
X-Received: by 2002:ac8:7d03:0:b0:305:2670:8a91 with SMTP id
g3-20020ac87d03000000b0030526708a91mr6596qtb.247.1654901733417; Fri, 10 Jun
2022 15:55:33 -0700 (PDT)
Path: i2pn2.org!i2pn.org!usenet.blueworldhosting.com!feed1.usenet.blueworldhosting.com!peer03.iad!feed-me.highwinds-media.com!news.highwinds-media.com!news-out.google.com!nntp.google.com!postnews.google.com!google-groups.googlegroups.com!not-for-mail
Newsgroups: sci.physics.relativity
Date: Fri, 10 Jun 2022 15:55:33 -0700 (PDT)
In-Reply-To: <lYCdnQd9Taw_Dz7_nZ2dnUU7_8zNnZ2d@giganews.com>
Injection-Info: google-groups.googlegroups.com; posting-host=2601:601:1700:7df0:e06d:7e32:7d75:63b7;
posting-account=mPYpNwoAAADYT6u25jo4wRqpXbzZAAhf
NNTP-Posting-Host: 2601:601:1700:7df0:e06d:7e32:7d75:63b7
References: <b067e4e7-bb62-4130-950c-5663f4983806n@googlegroups.com>
<5086313b-2d82-4e27-9aae-e1dd2cf7a954n@googlegroups.com> <60df787e-d27b-4f10-b7b2-03b3692ee97cn@googlegroups.com>
<_bmdnXI-c7KTIT3_nZ2dnUU7_8zNnZ2d@giganews.com> <1ccdd138-0756-49d9-b980-039996d3392fn@googlegroups.com>
<lYCdnQd9Taw_Dz7_nZ2dnUU7_8zNnZ2d@giganews.com>
User-Agent: G2/1.0
MIME-Version: 1.0
Message-ID: <65fdf573-e4c2-4974-8bf0-c241fa528606n@googlegroups.com>
Subject: Re: Special Relativity Fatal Error
From: fultonis...@gmail.com (Stan Fultoni)
Injection-Date: Fri, 10 Jun 2022 22:55:33 +0000
Content-Type: text/plain; charset="UTF-8"
Content-Transfer-Encoding: quoted-printable
X-Received-Bytes: 3182
 by: Stan Fultoni - Fri, 10 Jun 2022 22:55 UTC

On Friday, June 10, 2022 at 11:51:22 AM UTC-7, tjrob137 wrote:
> On 6/9/22 3:09 PM, Tom Capizzi wrote:
> > [...] Albert Einstein defined a legitimate indirect measurement as
> > one that produces the same value as a direct measurement would.
>
> What do you mean by that? What are "direct and indirect measurements"?

He's just referring to Einstein's popular booklet (Chapter 2) in which he first describes coordinate systems in terms of a grid of measuring rods, and then he says that, in practice, "the magnitudes of coordinates are not actually determined by constructions with rigid rods, by by indirect means. [Nevertheless, ] if the results of physics and astronomy are to maintain their clearness, the physics meaning of specifications of positions must always be sought in accordance with the above considerations". That's what Tom means when he paraphrases Einstein by saying that a legitimate indirect measurement is one that agrees with a direction measurement, i.e., by the grid of rulers.

The problem with Tom's beliefs is that, on one hand, he says he agrees that inertia-based coordinates are related by Lorentz transformations, but on the other hand, he disagrees with special relativity. He somehow doesn't grasp that those are one and the same thing. Notably, he insists (as do many here) that relativistic effects are not real, e.g., "nothing shrinks", because he (like many here) fails to grasp the distinction between passive and active transformations. This has been carefully explained to him (as it has to many others here) but he is simply incapable of grasping it (as are many here).

Re: Special Relativity Fatal Error

<970dd701-34f1-4e62-a8dc-3e0f1cafe448n@googlegroups.com>

  copy mid

https://www.novabbs.com/tech/article-flat.php?id=91646&group=sci.physics.relativity#91646

  copy link   Newsgroups: sci.physics.relativity
X-Received: by 2002:a05:622a:189f:b0:304:eb5d:5ca1 with SMTP id v31-20020a05622a189f00b00304eb5d5ca1mr24325086qtc.396.1654902386540;
Fri, 10 Jun 2022 16:06:26 -0700 (PDT)
X-Received: by 2002:a05:6214:5d06:b0:465:a991:3506 with SMTP id
me6-20020a0562145d0600b00465a9913506mr35971594qvb.112.1654902386325; Fri, 10
Jun 2022 16:06:26 -0700 (PDT)
Path: i2pn2.org!i2pn.org!usenet.blueworldhosting.com!feed1.usenet.blueworldhosting.com!peer03.iad!feed-me.highwinds-media.com!news.highwinds-media.com!news-out.google.com!nntp.google.com!postnews.google.com!google-groups.googlegroups.com!not-for-mail
Newsgroups: sci.physics.relativity
Date: Fri, 10 Jun 2022 16:06:26 -0700 (PDT)
In-Reply-To: <lYCdnQd9Taw_Dz7_nZ2dnUU7_8zNnZ2d@giganews.com>
Injection-Info: google-groups.googlegroups.com; posting-host=209.6.152.121; posting-account=anpm0goAAAD7eq4-R7Tlsnov4nyr6Xqb
NNTP-Posting-Host: 209.6.152.121
References: <b067e4e7-bb62-4130-950c-5663f4983806n@googlegroups.com>
<5086313b-2d82-4e27-9aae-e1dd2cf7a954n@googlegroups.com> <60df787e-d27b-4f10-b7b2-03b3692ee97cn@googlegroups.com>
<_bmdnXI-c7KTIT3_nZ2dnUU7_8zNnZ2d@giganews.com> <1ccdd138-0756-49d9-b980-039996d3392fn@googlegroups.com>
<lYCdnQd9Taw_Dz7_nZ2dnUU7_8zNnZ2d@giganews.com>
User-Agent: G2/1.0
MIME-Version: 1.0
Message-ID: <970dd701-34f1-4e62-a8dc-3e0f1cafe448n@googlegroups.com>
Subject: Re: Special Relativity Fatal Error
From: tgcapi...@gmail.com (Tom Capizzi)
Injection-Date: Fri, 10 Jun 2022 23:06:26 +0000
Content-Type: text/plain; charset="UTF-8"
Content-Transfer-Encoding: quoted-printable
X-Received-Bytes: 9206
 by: Tom Capizzi - Fri, 10 Jun 2022 23:06 UTC

On Friday, June 10, 2022 at 2:51:22 PM UTC-4, tjrob137 wrote:
> [I'll give you a chance to explain your personal vocabulary and describe
> your claims in terms other people can understand. If you can't do that,
> I'll abandon the thread as useless.]
>
> On 6/9/22 3:09 PM, Tom Capizzi wrote:
> > [...] Albert Einstein defined a legitimate indirect measurement as
> > one that produces the same value as a direct measurement would.
> What do you mean by that?
>
> What is "legitimate" and how did Einstein define it? (That's a word he
> never used AFAIK)
>
> What are "direct and indirect measurements"?
>
> What "same value"?
> > His own gedanken experiments, and every real experiment show that
> > there is no protocol for indirect measurement above some threshold
> > velocity that satisfies this requirement of validity.
> What is "indirect measurement"?
>
> What "threshold velocity"?
>
> What "requirement of validity"?
>
> > [...]
>
> Your prose is very dense in undefined words -- none of your phrases that
> I quoted above are standard, they are your personal vocabulary. Describe
> what you mean so others can understand, and we can go on to the rest of
> your post.
>
> Tom Roberts

Sorry. I'm trying to describe phenomena which do not have universally accepted labels. I use terms that fit the subject. In my book, there will be a glossary of terms. For now, I will answer questions as they arise. For starters, I refer you to Einstein's own writing. Why should I paraphrase it and get called out for using non-standard vocabulary? Specifically, in "On the Electrodynamics..." he wrote,
"In accordance with the principle of relativity the length to be discovered by
the operation (a)—we will call it “the length of the rod in the moving system”—
must be equal to the length l of the stationary rod"
"Current kinematics tacitly assumes that the lengths determined by these two
operations are precisely equal, or in other words, that a moving rigid body at
the epoch t may in geometrical respects be perfectly represented by the same
body at rest in a definite position."
He goes on,
"If a material point is at rest relatively to this system of co-ordinates, its
position can be defined relatively thereto by the employment of rigid standards
of measurement and the methods of Euclidean geometry, and can be expressed
in Cartesian co-ordinates."
In "RELATIVITY The Special and General Theory", he goes further,
"From this consideration we see that it will be
advantageous if, in the description of position, it
should be possible by means of numerical measures
to make ourselves independent of the existence of
marked positions (possessing names) on the rigid
body of reference. In the physics of measurement
this is attained by the application of the Cartesian
system of co-ordinates."

The third quote refers to a direct measurement. The "rigid standards of measurement" are the uniform rigid rods that are placed end to end and counted to yield a coordinate. Nothing out of the ordinary here. But it is not relativity. The interval, the observer and the measuring rods are all relatively stationary. This is a Newtonian scenario. But Newtonian scenarios include very small relative velocities, too, for which it is not possible to directly compare a rigid measuring rod and the moving interval. These I collectively call quasistatic. Others have objected because quasistatic has a definition in electromagnetic theory. Tough. Words have more than one meaning. Usually, the intended meaning comes from the context in which the word is used. Quasistatic fields are fields which vary with such low frequency that their contribution can be neglected, since time varying magnitudes produce non-zero derivatives, but at low enough frequency, the derivative field can be ignored compared to the source. I do not refer to frequency of variation, but to magnitude of relative velocity. For slow enough velocities, the ratio v/c becomes effectively zero, which it would be exactly if v=0. So it's almost static, and for all practical purposes be considered static. The point is, direct measurements are not possible with a moving target. Any method can be used to measure the moving object. If it produces the same measurement as a direct measurement, it is a legitimate indirect measurement.

The "same value" is referred to in the first quote. One is the length of the rod in a moving system, while the other is the length of the stationary rod. I use the term "threshold velocity" to distinguish between quasistatic and relativistic. Newtonian laws are valid in quasistatic frames, but not in relativistic ones. This is a somewhat arbitrary boundary, and it depends on the degree of precision of measurements. A measurement that is only accurate to 1 part in a thousand allows a higher threshold than one which is accurate to 1 part in a million. Point is, quasistatic frames use approximations, but relativistic frames use the precise version. But their algebraic complexity is not always necessary. As a ballpark reference, velocities for which sin(tilt) ≈ tilt are generally speaking, quasistatic. Basically, if sin(tilt) ≈ tilt, then v/c ≈ 0, and cos(tilt) ≈ 1. In the more general case, v/c = sin(tilt) and cos(tilt) = 1/γ, the reciprocal of the Lorentz factor. Last, the requirement of validity is contained in the 2nd quote. The indirect measurement produces the same value as would a direct measurement by a co-moving observer.
This last condition is where Einstein went wrong. His gedanken experiments, and every physical experiment involving velocities that are not quasistatic, all agree that measurements of length are contracted and measurements of time are dilated. These explicitly do not agree with direct measurements. In many cases, these indirect measurement procedures do produce correct measurements, if applied to a quasistatic frame. At relativistic velocities, they break down, just like the rest of Newtonian mechanics. So, Einstein's assertion that the lengths of the moving rod and the stationary rod are equal is not true, in general. It is only true in quasistatic frames. Since he builds special relativity on this false premise of measurement, it is illogical and invalid.

So, why does it work? Because in adopting a Newtonian protocol, he excluded the time dilation effects and length contraction effects that are inherent in the Euclidean dot product. So, in order to agree with the experimental evidence, he had to put the two properties back in, after wrongfully excluding them in the first place. That fixed the numbers, but it did not alter the contradiction. Physics was forced to deny objective reality and give every observer their own subjective reality independent of all the others. Then we are told to pay no attention to the man behind the curtain, and to accept the whole package, discarding common sense along the way.

If we apply the correct measurement protocol, it tells all observers to never expect to be able to measure more than a cosine projection, period. At low speeds, the cosine is 1, and the measurement is 100% real. At high speeds, the cosine is less than 1. When we actually do the experiment, we measure a cosine projection. The physics' relativistic correction factor is unity.. Special relativity evaporates. It's all geometry.

Re: Special Relativity Fatal Error

<5158559d-ba7c-4b7c-8255-e874f4f9182fn@googlegroups.com>

  copy mid

https://www.novabbs.com/tech/article-flat.php?id=91647&group=sci.physics.relativity#91647

  copy link   Newsgroups: sci.physics.relativity
X-Received: by 2002:a05:620a:4411:b0:6a7:6030:f9d1 with SMTP id v17-20020a05620a441100b006a76030f9d1mr1535044qkp.14.1654903814159;
Fri, 10 Jun 2022 16:30:14 -0700 (PDT)
X-Received: by 2002:a05:622a:20c:b0:304:f6db:6631 with SMTP id
b12-20020a05622a020c00b00304f6db6631mr19435672qtx.257.1654903813958; Fri, 10
Jun 2022 16:30:13 -0700 (PDT)
Path: i2pn2.org!i2pn.org!usenet.blueworldhosting.com!feed1.usenet.blueworldhosting.com!peer03.iad!feed-me.highwinds-media.com!news.highwinds-media.com!news-out.google.com!nntp.google.com!postnews.google.com!google-groups.googlegroups.com!not-for-mail
Newsgroups: sci.physics.relativity
Date: Fri, 10 Jun 2022 16:30:13 -0700 (PDT)
In-Reply-To: <65fdf573-e4c2-4974-8bf0-c241fa528606n@googlegroups.com>
Injection-Info: google-groups.googlegroups.com; posting-host=209.6.152.121; posting-account=anpm0goAAAD7eq4-R7Tlsnov4nyr6Xqb
NNTP-Posting-Host: 209.6.152.121
References: <b067e4e7-bb62-4130-950c-5663f4983806n@googlegroups.com>
<5086313b-2d82-4e27-9aae-e1dd2cf7a954n@googlegroups.com> <60df787e-d27b-4f10-b7b2-03b3692ee97cn@googlegroups.com>
<_bmdnXI-c7KTIT3_nZ2dnUU7_8zNnZ2d@giganews.com> <1ccdd138-0756-49d9-b980-039996d3392fn@googlegroups.com>
<lYCdnQd9Taw_Dz7_nZ2dnUU7_8zNnZ2d@giganews.com> <65fdf573-e4c2-4974-8bf0-c241fa528606n@googlegroups.com>
User-Agent: G2/1.0
MIME-Version: 1.0
Message-ID: <5158559d-ba7c-4b7c-8255-e874f4f9182fn@googlegroups.com>
Subject: Re: Special Relativity Fatal Error
From: tgcapi...@gmail.com (Tom Capizzi)
Injection-Date: Fri, 10 Jun 2022 23:30:14 +0000
Content-Type: text/plain; charset="UTF-8"
Content-Transfer-Encoding: quoted-printable
X-Received-Bytes: 4914
 by: Tom Capizzi - Fri, 10 Jun 2022 23:30 UTC

On Friday, June 10, 2022 at 6:55:34 PM UTC-4, Stan Fultoni wrote:
> On Friday, June 10, 2022 at 11:51:22 AM UTC-7, tjrob137 wrote:
> > On 6/9/22 3:09 PM, Tom Capizzi wrote:
> > > [...] Albert Einstein defined a legitimate indirect measurement as
> > > one that produces the same value as a direct measurement would.
> >
> > What do you mean by that? What are "direct and indirect measurements"?
>
> He's just referring to Einstein's popular booklet (Chapter 2) in which he first describes coordinate systems in terms of a grid of measuring rods, and then he says that, in practice, "the magnitudes of coordinates are not actually determined by constructions with rigid rods, by by indirect means. [Nevertheless, ] if the results of physics and astronomy are to maintain their clearness, the physics meaning of specifications of positions must always be sought in accordance with the above considerations". That's what Tom means when he paraphrases Einstein by saying that a legitimate indirect measurement is one that agrees with a direction measurement, i.e., by the grid of rulers.
>
> The problem with Tom's beliefs is that, on one hand, he says he agrees that inertia-based coordinates are related by Lorentz transformations, but on the other hand, he disagrees with special relativity. He somehow doesn't grasp that those are one and the same thing. Notably, he insists (as do many here) that relativistic effects are not real, e.g., "nothing shrinks", because he (like many here) fails to grasp the distinction between passive and active transformations. This has been carefully explained to him (as it has to many others here) but he is simply incapable of grasping it (as are many here).

Fake distinctions like active and passive transformations are a piss-poor argument. Sure, they are different, but so what? Measurement is about the difference between the endpoints. It does not matter what path they took in getting to their final position. All that matters is their positions relative to each other. And whether I observe a point being rotated by relative velocity or rotate the coordinate system underneath, "Note the equivalence between the two kinds of transformations: the coordinates of the new point in the active transformation and the new coordinates of the point in the passive transformation are the same, namely: (vX,vY,vZ) = (v'x,v'y,v'z)." Wikipedia

If there is any substance to your charge, then my argument should lead to a contradiction. Please identify it for us, or I will disregard your objections. By the way, the Lorentz transformation is not special relativity. It is a hyperbolic rotation and is well supported by hyperbolic trigonometry. Special relativity is a kludge that is based on false premises and corrupted logic. To say they are the same reveals your ignorance. And when I tell you that nothing shrinks, it has nothing to do with active or passive transformations. It has to do with the fact that there are more dimensions than Minkowski imagined, and when we take into consideration the projections in these extra dimensions NOTHING SHRINKS. Things rotate into the extra dimensions, where they cannot be seen or measured. How that has anything to do with active or passive transformation escapes me.

Re: Special Relativity Fatal Error

<549d06c9-fd3d-4a54-9e24-b5d8273d043fn@googlegroups.com>

  copy mid

https://www.novabbs.com/tech/article-flat.php?id=91648&group=sci.physics.relativity#91648

  copy link   Newsgroups: sci.physics.relativity
X-Received: by 2002:a05:6214:27c6:b0:467:dcbf:6442 with SMTP id ge6-20020a05621427c600b00467dcbf6442mr31763972qvb.24.1654904240590;
Fri, 10 Jun 2022 16:37:20 -0700 (PDT)
X-Received: by 2002:a05:622a:cb:b0:304:eb7b:5e11 with SMTP id
p11-20020a05622a00cb00b00304eb7b5e11mr24855168qtw.88.1654904240404; Fri, 10
Jun 2022 16:37:20 -0700 (PDT)
Path: i2pn2.org!i2pn.org!usenet.blueworldhosting.com!feed1.usenet.blueworldhosting.com!peer03.iad!feed-me.highwinds-media.com!news.highwinds-media.com!news-out.google.com!nntp.google.com!postnews.google.com!google-groups.googlegroups.com!not-for-mail
Newsgroups: sci.physics.relativity
Date: Fri, 10 Jun 2022 16:37:20 -0700 (PDT)
In-Reply-To: <5158559d-ba7c-4b7c-8255-e874f4f9182fn@googlegroups.com>
Injection-Info: google-groups.googlegroups.com; posting-host=2601:601:1700:7df0:e06d:7e32:7d75:63b7;
posting-account=mPYpNwoAAADYT6u25jo4wRqpXbzZAAhf
NNTP-Posting-Host: 2601:601:1700:7df0:e06d:7e32:7d75:63b7
References: <b067e4e7-bb62-4130-950c-5663f4983806n@googlegroups.com>
<5086313b-2d82-4e27-9aae-e1dd2cf7a954n@googlegroups.com> <60df787e-d27b-4f10-b7b2-03b3692ee97cn@googlegroups.com>
<_bmdnXI-c7KTIT3_nZ2dnUU7_8zNnZ2d@giganews.com> <1ccdd138-0756-49d9-b980-039996d3392fn@googlegroups.com>
<lYCdnQd9Taw_Dz7_nZ2dnUU7_8zNnZ2d@giganews.com> <65fdf573-e4c2-4974-8bf0-c241fa528606n@googlegroups.com>
<5158559d-ba7c-4b7c-8255-e874f4f9182fn@googlegroups.com>
User-Agent: G2/1.0
MIME-Version: 1.0
Message-ID: <549d06c9-fd3d-4a54-9e24-b5d8273d043fn@googlegroups.com>
Subject: Re: Special Relativity Fatal Error
From: fultonis...@gmail.com (Stan Fultoni)
Injection-Date: Fri, 10 Jun 2022 23:37:20 +0000
Content-Type: text/plain; charset="UTF-8"
Content-Transfer-Encoding: quoted-printable
X-Received-Bytes: 6579
 by: Stan Fultoni - Fri, 10 Jun 2022 23:37 UTC

On Thursday, June 9, 2022 at 9:25:53 PM UTC-7, tgca...@gmail.com wrote:
> > > I do not agree that physics has anything to do with it. It is purely mathematical.
> > Passive transformations are purely mathematical, but active transformations are not. It is the active transformations that give Lorentz invariance it's physical significance. Again, since you assure me that your argument leads to the same predictions of measurements as special relativity, you agree that all the laws of physics are Lorentz invariant, meaning that inertia-based coordinate systems are related by Lorentz transformations. Excellent.
> > > Relativity is the real projection of an invariant complex magnitude.
> > Again, you are talking only about passive transformations, which are pure math. Physics is about active transformations that reveal the Lorentz invariance of all physical laws.
> > > How you could assert "when in fact you actually agree with it completely" is beyond me.
> > That may be true, i.e., it may be beyond you. But based on your assurance that you agree with all the empirical manifestations of Lorentz invariance, you agree completely with special relativity. If you are determined not to agree, you need to retract your claim -- which then exposes you to empirical falsification.
>
> I agree that the results of measurements agree with the predictions of the Lorentz transform.

Presumably you mean transformation, not transform. That's great, because it signifies that you agree that the laws of physics are (locally) Lorentz invariant, and that inertia-based coordinate systems are related by Lorentz transformations, which is the entire content of special relativity.

> However, I reject the claim that anything physically shrinks.

Rather than playing semantic games, just be clear about what we mean: If, in terms of inertial coordinate system S, the simultaneous distance between the ends of a solid rod at rest in S is L, and if we then gently accelerate the rod and allow it to reach equilibrium moving at speed v, the simultaneous distance between the ends of the rod (still in terms of S) is now L sqrt(1-v^2/c^2). This is the prediction of special relativity, and you have said you agree with it, so you really have no point.

Remember, every school boy knows about the description in terms of hyperbolic rotation. Poincare and Minkowski emphasized that. This is entirely consistent with special relativity. Also, remember that Maxwell, Heaviside, and Searle already showed in the 1800's that Maxwell's equations imply that any object governed by electromagnetic interactions must shrink in the direction of motion by the factor sqrt(1-v^2/c^2). Since Maxwell's equations are Lorentz invariant, this shouldn't be surprising.

> It's a hyperbolic rotation, w. Nothing shrinks.

You're confusing passive and active transformations. An actual active acceleration of a material object is not identical to a simple hyperbolic rotation, it's more complicated, although at equilibrium for sufficiently gentle acceleration it asymptotically approaches it. This is all just Relativity 101.

> I don't accept your classification of active and passive transformations....

It isn't a classification, it's simply a statement of well known fact, clarifying an important distinction that often confuses beginners who wonder if relativistic effects are real.

> When special relativity restores objective reality...

Special relativity involves only a single objective reality. The notion that relativity is a subjectivist theory with different realities for different "observers" is just a common newbie fallacy. Remember, Minkowski called relativity "The Theory of the Absolute World", and Einstein thought it should be named "Invariant Theory". It was Planck who got people to call it relativity theory.

> and includes 4 complex coordinates instead of real scalars...

Many people (including Einstein) have attempted to generalize geometry to include four (or even more) complex dimensions, and of course the Hilbert space of quantum mechanics has complex dimensions (along with the measurement problem), but for the relations of space and time it has not been a fruitful idea. The spatio-temporal relations between events are stubbornly real, as are the eigenvalues of quantum mechanical operators.

> then we can talk about how much I agree with it. As it stands, special relativity is a fraud.

You contradict yourself. You've repeatedly stated you agree with special relativity completely, meaning you agree with all of its empirical predictions, which is all special relativity consists of. But now you say it is a fraud. One of the most important attributes of science is logical consistency. You've got much room for improvement in that department.

Re: Special Relativity Fatal Error

<cdc377c3-f408-433f-b88d-72006730e2abn@googlegroups.com>

  copy mid

https://www.novabbs.com/tech/article-flat.php?id=91649&group=sci.physics.relativity#91649

  copy link   Newsgroups: sci.physics.relativity
X-Received: by 2002:ac8:58cc:0:b0:305:1da9:7f5d with SMTP id u12-20020ac858cc000000b003051da97f5dmr4032032qta.345.1654908216601;
Fri, 10 Jun 2022 17:43:36 -0700 (PDT)
X-Received: by 2002:a05:620a:4590:b0:6a7:2543:2938 with SMTP id
bp16-20020a05620a459000b006a725432938mr8099992qkb.590.1654908216446; Fri, 10
Jun 2022 17:43:36 -0700 (PDT)
Path: i2pn2.org!i2pn.org!weretis.net!feeder8.news.weretis.net!news.szaf.org!fu-berlin.de!news-out.google.com!nntp.google.com!postnews.google.com!google-groups.googlegroups.com!not-for-mail
Newsgroups: sci.physics.relativity
Date: Fri, 10 Jun 2022 17:43:36 -0700 (PDT)
In-Reply-To: <5158559d-ba7c-4b7c-8255-e874f4f9182fn@googlegroups.com>
Injection-Info: google-groups.googlegroups.com; posting-host=2601:601:1700:7df0:e06d:7e32:7d75:63b7;
posting-account=mPYpNwoAAADYT6u25jo4wRqpXbzZAAhf
NNTP-Posting-Host: 2601:601:1700:7df0:e06d:7e32:7d75:63b7
References: <b067e4e7-bb62-4130-950c-5663f4983806n@googlegroups.com>
<5086313b-2d82-4e27-9aae-e1dd2cf7a954n@googlegroups.com> <60df787e-d27b-4f10-b7b2-03b3692ee97cn@googlegroups.com>
<_bmdnXI-c7KTIT3_nZ2dnUU7_8zNnZ2d@giganews.com> <1ccdd138-0756-49d9-b980-039996d3392fn@googlegroups.com>
<lYCdnQd9Taw_Dz7_nZ2dnUU7_8zNnZ2d@giganews.com> <65fdf573-e4c2-4974-8bf0-c241fa528606n@googlegroups.com>
<5158559d-ba7c-4b7c-8255-e874f4f9182fn@googlegroups.com>
User-Agent: G2/1.0
MIME-Version: 1.0
Message-ID: <cdc377c3-f408-433f-b88d-72006730e2abn@googlegroups.com>
Subject: Re: Special Relativity Fatal Error
From: fultonis...@gmail.com (Stan Fultoni)
Injection-Date: Sat, 11 Jun 2022 00:43:36 +0000
Content-Type: text/plain; charset="UTF-8"
Content-Transfer-Encoding: quoted-printable
 by: Stan Fultoni - Sat, 11 Jun 2022 00:43 UTC

On Friday, June 10, 2022 at 4:30:15 PM UTC-7, tgca...@gmail.com wrote:
> Fake distinctions like active and passive transformations are a piss-poor
> argument. Sure, they are different, but so what?

You contradict yourself. First you say it is a fake distinction, then you confess that it is a real distinction, but you don't grasp the significance of it. Those are two very different responses. A hallmark of science is logical consistency.

> Measurement is about the difference between the endpoints.

To be clear and correct, a measurement of a spatial distance between two entities in terms of a given system of coordinates x,t pertains to the spatial distance (i.e., difference in the space coordinates) between those two entities at the same time (i.e., equal time coordinates).

> Whether I observe a point being rotated by relative velocity or rotate the
> coordinate system underneath...

A "point" is point-like and hence cannot be rotated, so you must have been struggling to express either a point revolving about an axis or an extended object rotating. In either case, those are physically very different from simply describing things in terms of different systems of coordinates. Of course, we choose our passive transformations to match the effects of active transformations. This is what leads us to Lorentz invariance and special relativity. You've assured me that you agree with special relativity's predictions whole-heartedly, so you really have no point.

> If there is any substance to your charge, then my argument should lead to a contradiction.

You don't have an argument, you just have a senseless assertion that inertia-based coordinate systems are invalid for entities moving at high speed. That's just ridiculous.

> The Lorentz transformation is not special relativity.

The fact that inertia based coordinates are related by Lorentz transformations *is* special relativity.

> When I tell you that nothing shrinks, it has to do with the fact that there are more
> dimensions than Minkowski imagined...

That's a weird statement, because Minkowski agreed with everyone else that there are three space dimensions and one time dimension. How many do *you* think there are?

Re: Special Relativity Fatal Error

<be2df59e-1afe-4669-8021-9e31a5221ca1n@googlegroups.com>

  copy mid

https://www.novabbs.com/tech/article-flat.php?id=91652&group=sci.physics.relativity#91652

  copy link   Newsgroups: sci.physics.relativity
X-Received: by 2002:ac8:4e51:0:b0:305:754:9f99 with SMTP id e17-20020ac84e51000000b0030507549f99mr12900465qtw.684.1654918278772;
Fri, 10 Jun 2022 20:31:18 -0700 (PDT)
X-Received: by 2002:a05:620a:4249:b0:6a3:303b:6d26 with SMTP id
w9-20020a05620a424900b006a3303b6d26mr32726309qko.0.1654918278579; Fri, 10 Jun
2022 20:31:18 -0700 (PDT)
Path: i2pn2.org!i2pn.org!weretis.net!feeder8.news.weretis.net!proxad.net!feeder1-2.proxad.net!209.85.160.216.MISMATCH!news-out.google.com!nntp.google.com!postnews.google.com!google-groups.googlegroups.com!not-for-mail
Newsgroups: sci.physics.relativity
Date: Fri, 10 Jun 2022 20:31:18 -0700 (PDT)
In-Reply-To: <549d06c9-fd3d-4a54-9e24-b5d8273d043fn@googlegroups.com>
Injection-Info: google-groups.googlegroups.com; posting-host=209.6.152.121; posting-account=anpm0goAAAD7eq4-R7Tlsnov4nyr6Xqb
NNTP-Posting-Host: 209.6.152.121
References: <b067e4e7-bb62-4130-950c-5663f4983806n@googlegroups.com>
<5086313b-2d82-4e27-9aae-e1dd2cf7a954n@googlegroups.com> <60df787e-d27b-4f10-b7b2-03b3692ee97cn@googlegroups.com>
<_bmdnXI-c7KTIT3_nZ2dnUU7_8zNnZ2d@giganews.com> <1ccdd138-0756-49d9-b980-039996d3392fn@googlegroups.com>
<lYCdnQd9Taw_Dz7_nZ2dnUU7_8zNnZ2d@giganews.com> <65fdf573-e4c2-4974-8bf0-c241fa528606n@googlegroups.com>
<5158559d-ba7c-4b7c-8255-e874f4f9182fn@googlegroups.com> <549d06c9-fd3d-4a54-9e24-b5d8273d043fn@googlegroups.com>
User-Agent: G2/1.0
MIME-Version: 1.0
Message-ID: <be2df59e-1afe-4669-8021-9e31a5221ca1n@googlegroups.com>
Subject: Re: Special Relativity Fatal Error
From: tgcapi...@gmail.com (Tom Capizzi)
Injection-Date: Sat, 11 Jun 2022 03:31:18 +0000
Content-Type: text/plain; charset="UTF-8"
Content-Transfer-Encoding: quoted-printable
 by: Tom Capizzi - Sat, 11 Jun 2022 03:31 UTC

On Friday, June 10, 2022 at 7:37:22 PM UTC-4, Stan Fultoni wrote:
> On Thursday, June 9, 2022 at 9:25:53 PM UTC-7, tgca...@gmail.com wrote:
> > > > I do not agree that physics has anything to do with it. It is purely mathematical.
> > > Passive transformations are purely mathematical, but active transformations are not. It is the active transformations that give Lorentz invariance it's physical significance. Again, since you assure me that your argument leads to the same predictions of measurements as special relativity, you agree that all the laws of physics are Lorentz invariant, meaning that inertia-based coordinate systems are related by Lorentz transformations. Excellent.
> > > > Relativity is the real projection of an invariant complex magnitude..
> > > Again, you are talking only about passive transformations, which are pure math. Physics is about active transformations that reveal the Lorentz invariance of all physical laws.
> > > > How you could assert "when in fact you actually agree with it completely" is beyond me.
> > > That may be true, i.e., it may be beyond you. But based on your assurance that you agree with all the empirical manifestations of Lorentz invariance, you agree completely with special relativity. If you are determined not to agree, you need to retract your claim -- which then exposes you to empirical falsification.
> >
> > I agree that the results of measurements agree with the predictions of the Lorentz transform.
> Presumably you mean transformation, not transform. That's great, because it signifies that you agree that the laws of physics are (locally) Lorentz invariant, and that inertia-based coordinate systems are related by Lorentz transformations, which is the entire content of special relativity.
> > However, I reject the claim that anything physically shrinks.
> Rather than playing semantic games, just be clear about what we mean: If, in terms of inertial coordinate system S, the simultaneous distance between the ends of a solid rod at rest in S is L, and if we then gently accelerate the rod and allow it to reach equilibrium moving at speed v, the simultaneous distance between the ends of the rod (still in terms of S) is now L sqrt(1-v^2/c^2). This is the prediction of special relativity, and you have said you agree with it, so you really have no point.
>
> Remember, every school boy knows about the description in terms of hyperbolic rotation. Poincare and Minkowski emphasized that. This is entirely consistent with special relativity. Also, remember that Maxwell, Heaviside, and Searle already showed in the 1800's that Maxwell's equations imply that any object governed by electromagnetic interactions must shrink in the direction of motion by the factor sqrt(1-v^2/c^2). Since Maxwell's equations are Lorentz invariant, this shouldn't be surprising.
> > It's a hyperbolic rotation, w. Nothing shrinks.
> You're confusing passive and active transformations. An actual active acceleration of a material object is not identical to a simple hyperbolic rotation, it's more complicated, although at equilibrium for sufficiently gentle acceleration it asymptotically approaches it. This is all just Relativity 101.
> > I don't accept your classification of active and passive transformations...
> It isn't a classification, it's simply a statement of well known fact, clarifying an important distinction that often confuses beginners who wonder if relativistic effects are real.
> > When special relativity restores objective reality...
> Special relativity involves only a single objective reality. The notion that relativity is a subjectivist theory with different realities for different "observers" is just a common newbie fallacy. Remember, Minkowski called relativity "The Theory of the Absolute World", and Einstein thought it should be named "Invariant Theory". It was Planck who got people to call it relativity theory.
> > and includes 4 complex coordinates instead of real scalars...
> Many people (including Einstein) have attempted to generalize geometry to include four (or even more) complex dimensions, and of course the Hilbert space of quantum mechanics has complex dimensions (along with the measurement problem), but for the relations of space and time it has not been a fruitful idea. The spatio-temporal relations between events are stubbornly real, as are the eigenvalues of quantum mechanical operators.
> > then we can talk about how much I agree with it. As it stands, special relativity is a fraud.
> You contradict yourself. You've repeatedly stated you agree with special relativity completely, meaning you agree with all of its empirical predictions, which is all special relativity consists of. But now you say it is a fraud. One of the most important attributes of science is logical consistency. You've got much room for improvement in that department.

Special relativity is a fraud, not for what measurements it predicts but for the alleged reasons behind them. I can agree 100% with the results and still disagree about how they were obtained. As far as the length of a rod is concerned, measurement only accounts for the real component, the part we can observe and measure. There is more that we cannot see (but the co-moving observer can). The vector sum of what we can see and what we cannot see is the invariant length of the rod. Special relativity ignores the part we cannot see.
In an objective reality, all observers view the same scene. Einstein asserted that time dilation and length contraction were both physical. With two observers, one can argue that relative velocity does not identify which frame is actually moving, so the predictions might apply to either one. Since I can't identify which frame is moving, I can't argue that the prediction is wrong. Add a third observer at a different speed and the symmetry is broken. The principle of relativity says any of the three frames can be selected as the reference frame, whether it is moving or not. The problem is, regardless of which frame is chosen as reference, both of the other two frames will still disagree. So, which frame specifies the shrinkage correctly? If the effects were just illusions, then we could accept the ambiguity. But if one observer claims the shrinkage is x, and the other says it is y, explain how it can be possible for the same object to shrink different amounts, physically. Geometry says nothing shrinks, but velocity causes a rotation of rigid objects into dimensions outside the universe we can measure. There is no contradiction, because the measurement IS an illusion. Every observer is viewing the same object from a different angle, since velocity is a hyperbolic rotation. Your comment about "The spatio-temporal relations between events are stubbornly real" only confirms that we can only measure real projections. The imaginary projections cannot be seen or measured, but they exist, and they account for the shrunken real measurements of time and distance, as well as the excess relativistic momentum that Newton fails to account for. The insistence on a Newtonian measurement protocol results in these components being discarded. So relativity is about the relationships between real projections. What a surprise that "spatio-temporal relations between events are stubbornly real". We wear blinders to exclude the other components.
Finally, stating that I agree with the actual measurements of special relativity does not mean I accept your conclusion that I am being inconsistent. Einstein agreed with the measurements predicted by Lorentz Aether theory, but he did not support the aether. This is a well-known property of isomorphisms. They predict identical results, for completely different reasons. One can agree with the results without accepting the method by which they were determined. There is no experiment that can distinguish two isomorphisms, because they all get the same results. The things that make them different have no effect on the outcome of any experiment. You seem to specialize in creating strawmen and red herrings. Show me how using the Euclidean dot product as a measurement standard leads to a contradiction, and quit playing semantic games.

Re: Special Relativity Fatal Error

<9a15e598-b8f1-437d-904e-93aeb8f24319n@googlegroups.com>

  copy mid

https://www.novabbs.com/tech/article-flat.php?id=91653&group=sci.physics.relativity#91653

  copy link   Newsgroups: sci.physics.relativity
X-Received: by 2002:a05:622a:118b:b0:305:bc2:c7c9 with SMTP id m11-20020a05622a118b00b003050bc2c7c9mr10370378qtk.61.1654919195070;
Fri, 10 Jun 2022 20:46:35 -0700 (PDT)
X-Received: by 2002:a37:f609:0:b0:6a6:a6f7:3ead with SMTP id
y9-20020a37f609000000b006a6a6f73eadmr26565625qkj.501.1654919194905; Fri, 10
Jun 2022 20:46:34 -0700 (PDT)
Path: i2pn2.org!i2pn.org!usenet.blueworldhosting.com!feed1.usenet.blueworldhosting.com!peer01.iad!feed-me.highwinds-media.com!news.highwinds-media.com!news-out.google.com!nntp.google.com!postnews.google.com!google-groups.googlegroups.com!not-for-mail
Newsgroups: sci.physics.relativity
Date: Fri, 10 Jun 2022 20:46:34 -0700 (PDT)
In-Reply-To: <cdc377c3-f408-433f-b88d-72006730e2abn@googlegroups.com>
Injection-Info: google-groups.googlegroups.com; posting-host=209.6.152.121; posting-account=anpm0goAAAD7eq4-R7Tlsnov4nyr6Xqb
NNTP-Posting-Host: 209.6.152.121
References: <b067e4e7-bb62-4130-950c-5663f4983806n@googlegroups.com>
<5086313b-2d82-4e27-9aae-e1dd2cf7a954n@googlegroups.com> <60df787e-d27b-4f10-b7b2-03b3692ee97cn@googlegroups.com>
<_bmdnXI-c7KTIT3_nZ2dnUU7_8zNnZ2d@giganews.com> <1ccdd138-0756-49d9-b980-039996d3392fn@googlegroups.com>
<lYCdnQd9Taw_Dz7_nZ2dnUU7_8zNnZ2d@giganews.com> <65fdf573-e4c2-4974-8bf0-c241fa528606n@googlegroups.com>
<5158559d-ba7c-4b7c-8255-e874f4f9182fn@googlegroups.com> <cdc377c3-f408-433f-b88d-72006730e2abn@googlegroups.com>
User-Agent: G2/1.0
MIME-Version: 1.0
Message-ID: <9a15e598-b8f1-437d-904e-93aeb8f24319n@googlegroups.com>
Subject: Re: Special Relativity Fatal Error
From: tgcapi...@gmail.com (Tom Capizzi)
Injection-Date: Sat, 11 Jun 2022 03:46:35 +0000
Content-Type: text/plain; charset="UTF-8"
Content-Transfer-Encoding: quoted-printable
X-Received-Bytes: 5285
 by: Tom Capizzi - Sat, 11 Jun 2022 03:46 UTC

On Friday, June 10, 2022 at 8:43:38 PM UTC-4, Stan Fultoni wrote:
> On Friday, June 10, 2022 at 4:30:15 PM UTC-7, tgca...@gmail.com wrote:
> > Fake distinctions like active and passive transformations are a piss-poor
> > argument. Sure, they are different, but so what?
> You contradict yourself. First you say it is a fake distinction, then you confess that it is a real distinction, but you don't grasp the significance of it. Those are two very different responses. A hallmark of science is logical consistency.
> > Measurement is about the difference between the endpoints.
> To be clear and correct, a measurement of a spatial distance between two entities in terms of a given system of coordinates x,t pertains to the spatial distance (i.e., difference in the space coordinates) between those two entities at the same time (i.e., equal time coordinates).
>
> > Whether I observe a point being rotated by relative velocity or rotate the
> > coordinate system underneath...
>
> A "point" is point-like and hence cannot be rotated, so you must have been struggling to express either a point revolving about an axis or an extended object rotating. In either case, those are physically very different from simply describing things in terms of different systems of coordinates. Of course, we choose our passive transformations to match the effects of active transformations. This is what leads us to Lorentz invariance and special relativity. You've assured me that you agree with special relativity's predictions whole-heartedly, so you really have no point.
> > If there is any substance to your charge, then my argument should lead to a contradiction.
> You don't have an argument, you just have a senseless assertion that inertia-based coordinate systems are invalid for entities moving at high speed. That's just ridiculous.
>
> > The Lorentz transformation is not special relativity.
>
> The fact that inertia based coordinates are related by Lorentz transformations *is* special relativity.
>
> > When I tell you that nothing shrinks, it has to do with the fact that there are more
> > dimensions than Minkowski imagined...
>
> That's a weird statement, because Minkowski agreed with everyone else that there are three space dimensions and one time dimension. How many do *you* think there are?

Nit-picking again? So, maybe I should have said an "irrelevant" distinction.. Do you disagree that measurement is about the distance between endpoints, regardless of coordinate system orientation? Like Don Lincoln when I asked him point blank what happens to the meter stick when it is moving. He didn't answer my question. He took the meter stick out of the question and told me that the space between where the ends of the meterstick used to be was contracted. I wanted to know what happened when the meter stick was still in the picture. Point is, the space between the endpoints is a displacement. It has nothing to do with whether the coordinate system was rotated or what path the rod took to get to its final position. Coincidentally (?), the displacement is the cosine projection of distance, in general, and distance depends on the actual path between the initial and final positions. And the cosine projection of the distance is just the dot product of the path with the displacement vector.
So, forget everything else I wrote, except this. The correct relativistic protocol for measurement is the Euclidean dot product. Show me how that leads to a contradiction.

Re: Special Relativity Fatal Error

<82eebf64-648b-4db5-a33c-8f2353c74247n@googlegroups.com>

  copy mid

https://www.novabbs.com/tech/article-flat.php?id=91654&group=sci.physics.relativity#91654

  copy link   Newsgroups: sci.physics.relativity
X-Received: by 2002:a05:622a:58c:b0:305:79a:f53 with SMTP id c12-20020a05622a058c00b00305079a0f53mr13018082qtb.601.1654919830741;
Fri, 10 Jun 2022 20:57:10 -0700 (PDT)
X-Received: by 2002:a05:622a:48e:b0:2f9:26c6:d789 with SMTP id
p14-20020a05622a048e00b002f926c6d789mr40054930qtx.95.1654919830428; Fri, 10
Jun 2022 20:57:10 -0700 (PDT)
Path: i2pn2.org!i2pn.org!usenet.blueworldhosting.com!feed1.usenet.blueworldhosting.com!peer01.iad!feed-me.highwinds-media.com!news.highwinds-media.com!news-out.google.com!nntp.google.com!postnews.google.com!google-groups.googlegroups.com!not-for-mail
Newsgroups: sci.physics.relativity
Date: Fri, 10 Jun 2022 20:57:10 -0700 (PDT)
In-Reply-To: <cdc377c3-f408-433f-b88d-72006730e2abn@googlegroups.com>
Injection-Info: google-groups.googlegroups.com; posting-host=209.6.152.121; posting-account=anpm0goAAAD7eq4-R7Tlsnov4nyr6Xqb
NNTP-Posting-Host: 209.6.152.121
References: <b067e4e7-bb62-4130-950c-5663f4983806n@googlegroups.com>
<5086313b-2d82-4e27-9aae-e1dd2cf7a954n@googlegroups.com> <60df787e-d27b-4f10-b7b2-03b3692ee97cn@googlegroups.com>
<_bmdnXI-c7KTIT3_nZ2dnUU7_8zNnZ2d@giganews.com> <1ccdd138-0756-49d9-b980-039996d3392fn@googlegroups.com>
<lYCdnQd9Taw_Dz7_nZ2dnUU7_8zNnZ2d@giganews.com> <65fdf573-e4c2-4974-8bf0-c241fa528606n@googlegroups.com>
<5158559d-ba7c-4b7c-8255-e874f4f9182fn@googlegroups.com> <cdc377c3-f408-433f-b88d-72006730e2abn@googlegroups.com>
User-Agent: G2/1.0
MIME-Version: 1.0
Message-ID: <82eebf64-648b-4db5-a33c-8f2353c74247n@googlegroups.com>
Subject: Re: Special Relativity Fatal Error
From: tgcapi...@gmail.com (Tom Capizzi)
Injection-Date: Sat, 11 Jun 2022 03:57:10 +0000
Content-Type: text/plain; charset="UTF-8"
Content-Transfer-Encoding: quoted-printable
X-Received-Bytes: 4723
 by: Tom Capizzi - Sat, 11 Jun 2022 03:57 UTC

On Friday, June 10, 2022 at 8:43:38 PM UTC-4, Stan Fultoni wrote:
> On Friday, June 10, 2022 at 4:30:15 PM UTC-7, tgca...@gmail.com wrote:
> > Fake distinctions like active and passive transformations are a piss-poor
> > argument. Sure, they are different, but so what?
> You contradict yourself. First you say it is a fake distinction, then you confess that it is a real distinction, but you don't grasp the significance of it. Those are two very different responses. A hallmark of science is logical consistency.
> > Measurement is about the difference between the endpoints.
> To be clear and correct, a measurement of a spatial distance between two entities in terms of a given system of coordinates x,t pertains to the spatial distance (i.e., difference in the space coordinates) between those two entities at the same time (i.e., equal time coordinates).
>
> > Whether I observe a point being rotated by relative velocity or rotate the
> > coordinate system underneath...
>
> A "point" is point-like and hence cannot be rotated, so you must have been struggling to express either a point revolving about an axis or an extended object rotating. In either case, those are physically very different from simply describing things in terms of different systems of coordinates. Of course, we choose our passive transformations to match the effects of active transformations. This is what leads us to Lorentz invariance and special relativity. You've assured me that you agree with special relativity's predictions whole-heartedly, so you really have no point.
> > If there is any substance to your charge, then my argument should lead to a contradiction.
> You don't have an argument, you just have a senseless assertion that inertia-based coordinate systems are invalid for entities moving at high speed. That's just ridiculous.
>
> > The Lorentz transformation is not special relativity.
>
> The fact that inertia based coordinates are related by Lorentz transformations *is* special relativity.
>
> > When I tell you that nothing shrinks, it has to do with the fact that there are more
> > dimensions than Minkowski imagined...
>
> That's a weird statement, because Minkowski agreed with everyone else that there are three space dimensions and one time dimension. How many do *you* think there are?

Just noticed this last remark. In the first place, I thought Minkowski believed in 4 spacetime dimensions and it was Galileo and Newton who believed in the 3+1 model. I think the case is strong for each dimension to have a complex scalar coordinate. Does that count as 4 complex dimensions or 8 real coordinates? In any case, the magnitudes of the imaginary components are proportional to Proper velocity, so the number of non-zero coordinates depends on the orientation of the coordinate system relative to the velocity vector. The total magnitude of the Proper velocity is the same, no matter how many dimensions it projects into.

Re: Special Relativity Fatal Error

<jgil30Fbod1U1@mid.individual.net>

  copy mid

https://www.novabbs.com/tech/article-flat.php?id=91658&group=sci.physics.relativity#91658

  copy link   Newsgroups: sci.physics.relativity
Path: i2pn2.org!i2pn.org!weretis.net!feeder8.news.weretis.net!news.szaf.org!fu-berlin.de!uni-berlin.de!individual.net!not-for-mail
From: ttt_...@web.de (Thomas Heger)
Newsgroups: sci.physics.relativity
Subject: Re: Special Relativity Fatal Error
Date: Sat, 11 Jun 2022 06:47:33 +0200
Lines: 42
Message-ID: <jgil30Fbod1U1@mid.individual.net>
References: <b067e4e7-bb62-4130-950c-5663f4983806n@googlegroups.com> <5086313b-2d82-4e27-9aae-e1dd2cf7a954n@googlegroups.com> <60df787e-d27b-4f10-b7b2-03b3692ee97cn@googlegroups.com> <_bmdnXI-c7KTIT3_nZ2dnUU7_8zNnZ2d@giganews.com> <jgg33dFsidtU1@mid.individual.net> <e38eceea-33ff-467c-acfd-2a555eff781fn@googlegroups.com>
Mime-Version: 1.0
Content-Type: text/plain; charset=utf-8; format=flowed
Content-Transfer-Encoding: 7bit
X-Trace: individual.net t7+P43VCTZn4L7c+wxdA/w+gKtLSETaK7p5f5LppU2QroCeU+4
Cancel-Lock: sha1:ze0/ZnWifVe3gKQKCg7P6NTGp4o=
User-Agent: Mozilla/5.0 (Windows NT 6.0; WOW64; rv:31.0) Gecko/20100101 Thunderbird/31.4.0
In-Reply-To: <e38eceea-33ff-467c-acfd-2a555eff781fn@googlegroups.com>
 by: Thomas Heger - Sat, 11 Jun 2022 04:47 UTC

Am 10.06.2022 um 07:38 schrieb Maciej Wozniak:
> On Friday, 10 June 2022 at 07:28:18 UTC+2, Thomas Heger wrote:
>
>> Appartently the universe 'speaks' in a mathematical language, which is
>> also known as 'geometric algebra'.
>
> The universe is silent, like it always was. If you're
> applying some mathematical language to it - that's
> because you're trained to try.
>
>
No, the universe is not silent.

We see things all around us, which are somehow produced by the universe.

Now we could think, the universe itself would operate on a mathematical
foundation and acts like a large analoge computer.

This was more or less my own assumption, which I have used in my 'book'
about 'structured spacetime':

https://docs.google.com/presentation/d/1Ur3_giuk2l439fxUa8QHX4wTDxBEaM6lOlgVUa0cFU4/edit?usp=sharing

This concept works quite well and was based on the assumption, that the
universe itself uses a certain kind of math, by which pointlike elements
of spacetime are interconnected.

This would create certain patterns, which we call 'matter' in case of
they are timelike stable.

It is actually a relatively simple concept, streight forward and based
on a small set of plausible assumptions.

The only problem is, that these assumptions are not very intuitive and
difficult to understand.

TH

Re: Special Relativity Fatal Error

<33c1ca8d-cca7-44f1-bd35-198c03002564n@googlegroups.com>

  copy mid

https://www.novabbs.com/tech/article-flat.php?id=91659&group=sci.physics.relativity#91659

  copy link   Newsgroups: sci.physics.relativity
X-Received: by 2002:ae9:ef47:0:b0:6a6:809a:aab5 with SMTP id d68-20020ae9ef47000000b006a6809aaab5mr30824291qkg.615.1654922865078;
Fri, 10 Jun 2022 21:47:45 -0700 (PDT)
X-Received: by 2002:a05:620a:408e:b0:6a7:1815:a431 with SMTP id
f14-20020a05620a408e00b006a71815a431mr10730682qko.551.1654922864902; Fri, 10
Jun 2022 21:47:44 -0700 (PDT)
Path: i2pn2.org!i2pn.org!usenet.blueworldhosting.com!feed1.usenet.blueworldhosting.com!peer01.iad!feed-me.highwinds-media.com!news.highwinds-media.com!news-out.google.com!nntp.google.com!postnews.google.com!google-groups.googlegroups.com!not-for-mail
Newsgroups: sci.physics.relativity
Date: Fri, 10 Jun 2022 21:47:44 -0700 (PDT)
In-Reply-To: <9a15e598-b8f1-437d-904e-93aeb8f24319n@googlegroups.com>
Injection-Info: google-groups.googlegroups.com; posting-host=2601:601:1700:7df0:e06d:7e32:7d75:63b7;
posting-account=mPYpNwoAAADYT6u25jo4wRqpXbzZAAhf
NNTP-Posting-Host: 2601:601:1700:7df0:e06d:7e32:7d75:63b7
References: <b067e4e7-bb62-4130-950c-5663f4983806n@googlegroups.com>
<5086313b-2d82-4e27-9aae-e1dd2cf7a954n@googlegroups.com> <60df787e-d27b-4f10-b7b2-03b3692ee97cn@googlegroups.com>
<_bmdnXI-c7KTIT3_nZ2dnUU7_8zNnZ2d@giganews.com> <1ccdd138-0756-49d9-b980-039996d3392fn@googlegroups.com>
<lYCdnQd9Taw_Dz7_nZ2dnUU7_8zNnZ2d@giganews.com> <65fdf573-e4c2-4974-8bf0-c241fa528606n@googlegroups.com>
<5158559d-ba7c-4b7c-8255-e874f4f9182fn@googlegroups.com> <cdc377c3-f408-433f-b88d-72006730e2abn@googlegroups.com>
<9a15e598-b8f1-437d-904e-93aeb8f24319n@googlegroups.com>
User-Agent: G2/1.0
MIME-Version: 1.0
Message-ID: <33c1ca8d-cca7-44f1-bd35-198c03002564n@googlegroups.com>
Subject: Re: Special Relativity Fatal Error
From: fultonis...@gmail.com (Stan Fultoni)
Injection-Date: Sat, 11 Jun 2022 04:47:45 +0000
Content-Type: text/plain; charset="UTF-8"
Content-Transfer-Encoding: quoted-printable
X-Received-Bytes: 6519
 by: Stan Fultoni - Sat, 11 Jun 2022 04:47 UTC

On Friday, June 10, 2022 at 8:46:36 PM UTC-7, tgca...@gmail.com wrote:
> Do you disagree that measurement is about the distance between endpoints,
> regardless of coordinate system orientation?

A measurement of Q "is about" a measurement of Q. Duh. For example, a measurement of a spatial distance between two entities in terms of a given system of coordinates x,t is about the spatial distance (i.e., difference in the space coordinates) between those two entities at the same time (i.e., equal time coordinates).

> What happens to the meter stick when it is moving?

Again, you need to make the crucial distinction between passive and active transformations, and when you use the phrase "what happens to" you need to grasp the distinction between the concepts of (1) acceleration and (2) changing coordinate systems, as opposed to simply being in some state of uniform motion. We covered all this this before. Remember Maxwell's equations?

> The displacement is the cosine projection of distance...

What displacement, and what distance, and what projection? You're just mindlessly stringing words together, with no rational content.

> the cosine projection of the distance is just the dot product of the path with
> the displacement vector.

What distance, and what displacement vector, and what projection? Again, you're just mindlessly stringing words together.

> Forget everything else I wrote, except this. The correct relativistic protocol for
> measurement is the Euclidean dot product.

Look, physical phenomena can be described in terms of any system of coordinates we like, and it so happens that there is a special class of coordinate systems -- the inertia-based systems -- in terms of which the equations of physical laws take a uniquely simple homogeneous and isotropic form. Those are operationally established by constructing an orthogonal grid of standard rulers with clocks at each node, all at rest and inertially synchronized. Now, it can be shown that if every quantity of energy E has inertia E/c^2, then any two inertia-based coordinate systems are related by a Lorentz transformation, and this entails all the well-known relativistic effects. Is there something you think is wrong or unclear about this?

> ...measurement only accounts for the real component, the part we can observe and
> measure. There is more that we cannot see (but the co-moving observer can).

No, relativity is not a subjectivist theory... there are not different realities for different observers, there are merely different descriptions in terms of different systems of coordinates. And the energy of inertia implies that the laws of physics are locally Lorentz invariant. That is all.

> Einstein agreed with the measurements predicted by Lorentz Aether theory, but
> he did not support the aether.

It is well known that there is a Lorentzian interpretation of special relativity. It is not a different theory, it is just a different vocabulary for describing the same theory.

> They predict identical results, for completely different reasons.

Not true, they are entirely equivalent in every way except for descriptive vocabulary.

> Show me how using the Euclidean dot product as a measurement standard leads to
> a contradiction, and quit playing semantic games.

You have not explained what you mean by "using a dot product as a measurement standard". A spatio-temporal measurement standard is ordinarily understood to be a coordinate system, and it makes no sense to say a dot product is a coordinate system. They are different categories of things. If you are trying to say that the manifold of spatio-temporal relations has a Euclidean metric, then that is easily shown to be wrong. Is that what you are trying to claim?

> I thought Minkowski believed in 4 spacetime dimensions and it was Galileo and
> Newton who believed in the 3+1 model.

Nope, everyone from pre-historic times until today has recognized that there are three spatial dimensions and one time dimension. The difference between Newtonian space-time and the space-time of special relativity is just due to the fact that we now know that energy E has inertia E/c^2, so the manifold of spatio-temporal relations has a Minkowski pseudo-metric, represented by what you could call the "dot product", i.e., the invariant line element is given by the Lorentzian inner product.

> I think the case is strong for each dimension to have a complex scalar coordinate.

Nope, all evidence indicates conclusively that the coordinates are real-valued.

Pages:12345
server_pubkey.txt

rocksolid light 0.9.8
clearnet tor